Cardiology Passpoint

Ace your homework & exams now with Quizwiz!

A nurse is caring for a client with heart failure. The nurse knows that the client has left-sided heart failure when the client makes which statement?

"I sleep on three pillows each night." Explanation: Orthopnea is a classic sign of left-sided heart failure. The client commonly sleeps on several pillows at night to help facilitate breathing. Swollen feet, ascites, and anorexia are signs of right-sided heart failure.

The nurse evaluates the client's understanding of nutritional modifications to manage hypertension. The nurse knows the teaching was successful when the client makes what statement?

"Limiting my salt intake to 2 grams per day will lower my blood pressure." Explanation: To lower blood pressure, a client should limit daily salt intake to 2 g or less. Alcohol intake is associated with a higher incidence of hypertension, poor compliance with treatment, and refractory hypertension. Chronic, moderate caffeine intake and fat intake do not affect blood pressure.

Which client statement should the nurse evaluate as indicating the client's correct understanding of the causes of coronary artery disease (CAD)?

"The leading cause of CAD is atherosclerosis." Explanation: Atherosclerosis (plaque formation) is the leading cause of CAD. Cigarette smoking is the leading cause of lung cancer. Telling the client to ask the healthcare provider is not appropriate.

The nurse is preparing to administer 0.1 mg of digoxin intravenously. Digoxin comes in a concentration of 0.5 mg/2 ml. How many milliliters should the nurse administer? Record your answer using one decimal place.

0.4 Explanation: The nurse should administer 0.4 ml to administer 0.1 mg of digoxin I.V. if it comes in a concentration of 0.5 mg/2 ml, or 0.25 mg/ml.

A client with hypertensive emergency is being treated with sodium nitroprusside. In a dilution of 50 mg/250 mL, how many micrograms of nitroprusside are in each milliliter? Record your answer using a whole number.

200 Explanation: First, calculate the number of milligrams per milliliter: 50 mg/250 mL = 1 mg/5 mL = 0.2 mg/1 mL Next, calculate the number of micrograms in each milligram: 0.2 mg × 1,000 mcg = 200 mcg

The rapid response team has been called to manage an unwitnessed cardiac arrest in a client's hospital room. How long should the nurse estimate the maximum time a person can be without cardiopulmonary function and still not experience permanent brain damage?

4 to 6 minutes Explanation: After a person is without cardiopulmonary function for 4 to 6 minutes, permanent brain damage is almost certain. To prevent permanent brain damage, it is important to begin cardiopulmonary resuscitation promptly after a cardiopulmonary arrest.

The nurse observes a sudden dampening of the arterial waveform. What is the priority action by the nurse?

Assess the client's blood pressure. Explanation: The priority action is to assess the client. A hypotensive crisis can look like dampened waveform and be life threatening if not treated appropriately. After the client is assessed, the healthcare provider can be contacted for additional interventions including a fluid challenge. Changing the tubing and flushing the line can be completed after the client is assessed.

A client with chest pain is prescribed intravenous nitroglycerin. Which finding is of greatest concern for the nurse initiating the nitroglycerin drip?

Blood pressure is 88/46 mm Hg. Explanation: Nitroglycerin is a vasodilator that will lower blood pressure. The client is having chest pain, and the ST elevation indicates injury to the myocardium, which may benefit from nitroglycerin. The potassium and heart rate are within normal range.

The nurse in the emergency department (ED) is caring for a 73-year-old male client with a history of chronic obstructive pulmonary disease (COPD) who presents with severe shortness of breath and new-onset pedal edema. For each assessment finding below, click to specify if the finding is consistent with the disease process of cor pulmonale, asthma, or pulmonary embolism. Each finding may be consistent with more than 1 disease process.

Cor Pulmonale Tachypnea Dyspnea JV distention Wheezing Pedal Edema RUQ discomfort Asthma Tachypnea Dyspnea Wheezing PE Tachypnea Dyspnea JV distention

A client with a cerebral embolus is receiving IV recombinant tissue-type plasminogen activator (rt-PA). The nurse should evaluate the client for which expected therapeutic outcomes of this drug therapy?

Dissolved emboli Explanation: Thrombolytic agents such as alteplase are used for clients with a history of thrombus formation, cerebrovascular accidents, and chronic atrial fibrillation. The thrombolytic agents act by dissolving emboli. Thrombolytic agents do not directly improve perfusion or improve cerebral vascularization, nor do they prevent cerebral hemorrhage.

The nurse is discussing medications with a client with hypertension who has a prescription for furosemide daily. Which comment by the client indicates the client needs further education?

Explanation: Furosemide is a diuretic often prescribed for clients with hypertension or heart failure; the drug should not affect a client's ability to drive safely. Furosemide may cause orthostatic hypotension, and clients should be instructed to be careful when changing from supine to sitting to standing position. Diuretics should be taken in the morning if possible to prevent sleep disturbance due to the need to get up to void. Furosemide is a loop diuretic that is not potassium sparing; clients should take potassium supplements as prescribed and have their serum potassium levels checked at prescribed intervals.

A nurse is reviewing laboratory values for a client diagnosed with hyperlipidemia 6 months ago. Which results indicate that the client has been following their therapeutic regimen?

High density lipoproteins (HDL) increase from 25 mg/dl (0.65 mmol/L) to 40 mg/dl (1.03 mmol/L). Explanation: The goal of treating hyperlipidemia is to decrease total cholesterol and LDL levels while increasing HDL levels. HDL levels should be greater than 35 mg/dl. This client's increased HDL levels indicate that the client followed the therapeutic regimen. Recommended total cholesterol levels are below 200 mg/dl. LDL levels should be less than 160 mg/dl, or, in clients with known coronary artery disease (CAD) or diabetes mellitus, less than 70 mg/dl. Triglyceride levels should be between 100 and 200 mg/d.

The nurse on a medical unit is caring for a 61-year-old male client with multiple myeloma who presents with coughing, fever, and chills. The nurse is reviewing the Nurse's Notes and Laboratory Results of 0900 of day 1 and 1200 of day 3 of hospitalization with the provider to determine the client's readiness for discharge. For each assessment finding, click to specify if the finding indicates that the client's condition has improved, has not changed, or has declined.

Improved Breath sounds, O2 sat, HR,RR, cough, Declined AB assessment, calcium No change Creatinine

A client is admitted to the hospital with a diagnosis of suspected pulmonary embolism. Prescriptions include oxygen 2 to 4 L/min per nasal cannula, oximetry at all times, and IV administration of 5% dextrose in water at 100 mL/h. The client has increasing dyspnea and has a respiratory rate of 32 breaths/minute. The oxygen flow rate is set at 2 L/min. What should the nurse do first?

Increase the oxygen flow rate from 2 to 4 L/min. Explanation: The first action is to increase the oxygen flow rate from 2 to 4 L/min to help ensure adequate oxygenation for the client. Although it is important to notify the HCP for additional prescriptions and to obtain further assessment data, such as arterial blood gas measurements, it is a priority to support the client's cardiopulmonary system. It would be appropriate to reassure the client while these other interventions are occurring.

Which of the following explains the influence of aging on the development of peripheral vascular disease?

Increased resistance. Explanation: As people age, the accumulation of collagen in the intima of the blood vessels results in the vessels' becoming stiff and less flexible. Consequently, there is an increased resistance within the aging adult's circulatory system.

A client with a history of coronary artery disease (CAD) has been diagnosed with peripheral arterial disease. The health care provider (HCP) started the client on pentoxifylline once daily. Approximately 1 hour after receiving the initial dose of pentoxifylline, the client reports having chest pain. The nurse should first:

Inform the HCP. Explanation: Angina is an adverse reaction to pentoxifylline, which should be used cautiously in clients with CAD. The nurse should report the client's symptoms to the HCP , who may prescribe nitroglycerin and possibly discontinue the pentoxifylline. The client should rest until the chest pain subsides. It is not necessary at this point to initiate the rapid response team or start an intravenous infusion. The client's reports of symptoms should never be dismissed.

The nurse on a medical unit is caring for a 61-year-old male client with multiple myeloma who presents with coughing, fever, and chills. The nurse has received orders from the health care provider and is implementing the plan of care. Which intervention(s) will the nurse implement? Select all that apply.

Instruct the client to avoid crowds. Teach the client to wash hands frequently. Teach the client to perform cough and deep-breathing exercises. Encourage increased fluid intake.

A client is talking with the nurse about unsightly varicose veins and their discomfort. What information should the nurse provide to the client?

Keep the legs elevated when sitting or lying down. Explanation: The nurse instructs the client to elevate the legs to improve venous return and alleviate discomfort. Walking is encouraged to increase venous return. Sclerotherapy or laser treatment is done for cosmetic reasons, but it does not improve circulation. Surgery may be performed for severe venous insufficiency or recurrent thrombophlebitis in the varicosities. Femoral-popliteal bypass graft is a surgical intervention for arterial disease.

The nurse is reviewing the electrocardiogram of a client who has elevated ST segments visible in leads II, III, and aVf. Which choice is the nurse's best action?

Notify the healthcare provider. Explanation: Leads II, III, and aVF record electrical events on the inferior surface of the left ventricle; elevated ST segments indicate that the client is experiencing a myocardial infarction. The healthcare provider should be notified. Teaching should be delayed until the client is stable. An irregular heart rhythm that varies with respiration-sinus arrhythmia-is a normal variation of sinus rhythm; there is no intervention needed.

Which is the most important initial postprocedure nursing assessment for a client who has had a cardiac catheterization?

Observe the puncture site for swelling and bleeding. Explanation: Assessment of circulatory status, including observation of the puncture site, is of primary importance after a cardiac catheterization. Laboratory values and skin warmth and turgor are important to monitor but are not the most important initial nursing assessment. Neurologic assessment every 15 minutes is not required.

A client in the intensive care unit has an arterial line that reads 58/30 mm Hg on the monitor. What is the nurse's first action?

Obtain a manual blood pressure. Explanation: The first action would be to assess the blood pressure manually. If the A-Line is accurate, the client should be placed in the Trendelenburg position. If it is not accurate, the arterial line should be flushed and recalibrated.

A nurse hears an irregular heart rate of 110 bpm when listening to a client's chest. After assessing the client and noting the new onset of shortness of breath, which action should the nurse take next?

Obtain a prescription for a stat electrocardiogram. Explanation: The nurse should contact the health care provider to request a stat electrocardiogram to verify the change in rhythm. The cardiac rhythm and fast heart rate may predispose the client to decreased cardiac output. Administering an anti-anxiety medication may calm the client but does not effectively treat the problem of an irregular rhythm. Rechecking the pulse later leaves the problem unaddressed. A chest X-ray may be helpful, but it is not the priority at this time.

The nurse on a medical unit is caring for a 61-year-old male client with multiple myeloma who presents with coughing, fever, and chills. For each assessment finding, click to specify if the finding indicates pneumonia, pleuritis, or myocarditis.

Pneumonia Cough Pleural friction rub Fever tachypnea Crackles Sputum production Pleuritis Pleural friction rub tachypnea fever Myocarditis Fever tachypnea

A client with deep vein thrombosis has been receiving warfarin for 2 months. The client is to go to an anticoagulant monitoring laboratory every 3 weeks. The last visit to the laboratory was 2 weeks ago. The client reports bleeding gums, increased bruising, and dark stools. What should the nurse should instruct the client to do?

Return to laboratory for analysis of prothrombin times. Explanation: These symptoms suggest that the client is receiving too much warfarin; the client should return to the laboratory and have a blood sample drawn to determine the prothrombin levels and have the dosage of warfarin adjusted. The diet can influence clotting, but the client needs to first have the prothrombin levels checked. It is not necessary to contact the HCP; the client should return to the laboratory first, and the results of the prothrombin time will be reported to the HCP.

One goal in caring for a client with arterial occlusive disease is to promote vasodilation in the affected extremity. What should the nurse instruct the client to do to achieve this goal?

Stop smoking. Explanation: Nicotine causes vasospasm and impedes blood flow. Stopping smoking is the most significant lifestyle change the client can make. The client should eat low-fat foods as part of a balanced diet. The legs should not be elevated above the heart because this will impede arterial flow. The legs should be in a slightly dependent position. Jogging is not necessary and probably is not possible for many clients with arterial occlusive disease. A rehabilitation program that includes daily walking is suggested.

Following diagnosis of angina pectoris, a client reports being unable to walk up two flights of stairs without pain. What should the nurse instruct the client to do?

Take a nitroglycerin tablet before climbing the stairs. Explanation: Nitroglycerin may be used prophylactically before stressful physical activities such as stair climbing to help the client remain pain free. Climbing the stairs early in the day would have no impact on decreasing pain episodes. Resting before or after an activity is not as likely to help prevent an activity-related pain episode.

A client arrives in the emergency department reporting intense pain in the abdomen and tells the nurse that it feels like a heartbeat in the abdomen. Which nursing assessment would indicate potential rupture of an aortic aneurysm?

The client reports increasing severe back pain. Explanation: Increased severe back pain and increased irritation to nerves are indicative of a potential rupture of an aneurysm. The client would be hypertensive and present with tachycardia, so the other choices are not correct. Nausea, although possible, or a missed dose of medication, do not indicate potential rupture.

The client who had a permanent pacemaker implanted 2 days earlier is being discharged from the hospital. What evidence will indicate to the nurse that the client understands the discharge plan?

The client verbalizes safety precautions needed to prevent pacemaker malfunction. Explanation: Education is a major component of the discharge plan for a client with an artificial pacemaker. The client with a permanent pacemaker needs to be able to state specific information about safety precautions, such as to refrain from lifting more than 3 lb (1.35 kg) or stretching and bending. The client should know how to count the pulse and do so daily or as instructed by the health care provider (HCP). The client will not necessarily be placed on a low cholesterol diet. The client should resume activities, and does not need to remain on bed rest. The client should know signs and symptoms of a MI, but is not at risk because of the pacemaker.

The nurse has reviewed the Nurse's Notes and orders from 0930 and is implementing the plan of care with this client.

Which action(s) will the nurse take? Select all that apply. Infuse potassium chloride using an infusion pump. Teach the client signs and symptoms of digoxin toxicity to report to the provider. Review the daily schedule with the client to determine the optimal timing of the diuretic. Instruct the client to contact the provider before taking any herbals, supplements, or over-the-counter medications. Educate the client about increasing intake of potassium-rich foods. Encourage the client to use one pharmacy to fill and renew prescriptions.

An obese white male client, age 49, is diagnosed with hypercholesterolemia. The physician orders a low-fat, low-cholesterol, low-calorie diet to reduce blood lipid levels and promote weight loss. This diet is crucial to the client's well-being because his race, sex, and age increase his risk for coronary artery disease (CAD). To determine whether the client has other major risk factors for CAD, the nurse should assess for

a history of diabetes mellitus. Explanation: Diabetes mellitus, smoking, and hypertension are other major risk factors for CAD. Elevated HDL levels aren't a risk factor for CAD; in fact, increased HDL levels seem to protect against CAD. Ischemic heart disease is another term for CAD, not a risk factor. Alcoholism hasn't been identified as a major risk factor for CAD.

Which assessment finding supports the administration of protamine sulfate?

aPTT 3.5-5 times normal Explanation: Protamine sulfate is the antidote specific to heparin. The RBC, and platelet levels are normal. Normal aPTT in heparinized clients is 2-2.5 times normal. INR measurement relates to therapy with warfarin, not heparin. An INR value of 8 is abnormally high and would likely require administration of vitamin K, the antidote for warfarin.

When teaching a client with newly diagnosed hypertension about the pathophysiology of this disease, the nurse states that arterial baroreceptors, which monitor arterial pressure, are located in the carotid sinus. Which other area should the nurse mention as a site of arterial baroreceptors?

aorta Explanation: Arterial baroreceptors are located in the carotid sinus and aorta. There aren't any baroreceptors in the brachial artery, radial artery, or right ventricular wall.

The nurse is concerned about the risks of hypoxemia and metabolic acidosis in a client who is in shock. What finding should the nurse analyze for evidence of hypoxemia and metabolic acidosis in a client with shock?

arterial blood gas (ABG) findings Explanation: Analysis of ABG findings is essential for evidence of hypoxemia and metabolic acidosis. Low RBCs and hemoglobin correlate with hypovolemic shock and can lead to poor oxygenation. An elevated white blood cell count supports septic shock. Oxygen saturation levels are usually affected by hypoxemia but cannot be used to diagnose acid-base imbalances such as metabolic acidosis.

The nurse is assessing a client with chronic bronchitis. For which finding should the nurse suspect that the client is developing right-sided heart failure?

bilateral edema of the feet and ankles Explanation: A client with chronic bronchitis, a form of chronic obstructive pulmonary disease (COPD), may experience symptoms that are similar to those of left-sided heart failure, such as dyspnea on exertion. However, without other risk factors, the client with COPD is at risk for right-sided, not left-sided, heart failure. Bilateral edema of the feet and ankles would not occur with chronic bronchitis but is evidence of right-sided heart failure due to the resistance to venous return to the right side of the heart. Bilateral crackles that clear with coughing would occur with chronic bronchitis. Note that pulmonary edema is not expected with right-sided heart failure. Nail clubbing develops in chronic bronchitis because of chronic oxygen deprivation and is not evidence of heart failure.

The nurse has given a client a nitroglycerin tablet sublingually for angina. Which vital signs should be assessed following administration of nitroglycerin?

blood pressure Explanation: Nitroglycerin can cause hypotension. A priority nursing assessment after the administration of nitroglycerin is the client's blood pressure.Oxygen saturation, respiratory rate, and pulse rate are not priority nursing assessments after the administration of nitroglycerin.

A client is receiving nitroglycerin ointment to treat angina pectoris. The nurse evaluates the therapeutic effectiveness of this drug by assessing the client's response and checking for adverse effects. Which vital sign is most likely to reflect an adverse effect of nitroglycerin?

blood pressure 84/52 mm Hg Explanation: Hypotension and headache are the most common adverse effects of nitroglycerin. Therefore, blood pressure is the vital sign most likely to reflect an adverse effect of this drug. The nurse should check the client's blood pressure 1 hour after administering nitroglycerin ointment. A blood pressure decrease of 10 mm Hg is within the therapeutic range. If blood pressure falls more than 20 mm Hg below baseline, the nurse should remove the ointment and report the finding to the physician immediately. An above-normal heart rate (tachycardia) is a less common adverse effect of nitroglycerin. Respiratory rate and temperature don't change significantly after nitroglycerin administration.

A client is in hypovolemic shock. To determine the effectiveness of fluid replacement therapy, the nurse should monitor the client's

blood pressure. Explanation: With adequate fluid replacement, fluid volume in the intravascular space expands, raising the client's blood pressure. The hemoglobin level reflects red blood cell concentration, not overall fluid status. Temperature and heart rate aren't directly related to fluid status.

A client comes to the clinic with back pain that has been unrelieved by continuous ibuprofen use over the past several days. Current prescription medications include captopril and hydrochlorothiazide. Which laboratory value should the nurse address?

blood urea nitrogen (BUN) of 26 mg/dL and serum creatinine of 2.35 mg/dL Explanation: Nonsteroidal anti-inflammatory drugs (NSAIDs) can decrease the antihypertensive effect of ACE inhibitors and predispose clients to the development of acute renal failure. Common lab tests used to evaluate how well the kidneys are working are BUN, creatinine, and creatinine clearance. Labs such as sodium, potassium, CPK, and WBC levels will not provide information on renal function.

A nurse is preparing a teaching plan for a male client newly prescribed atenolol. Which information is important for the nurse to teach this client?

causes and treatments for erectile dysfunction Explanation: Erectile dysfunction is a potential adverse effect of beta blockers. The other symptoms are not side effects of this drug.

The nurse is assessing a client with irreversible shock. The nurse should document the progression of which expected finding?

circulatory collapse Explanation: Severe hypoperfusion to all vital organs results in failure of the vital functions and then circulatory collapse. Hypotension, anuria, respiratory distress, and acidosis are other symptoms associated with irreversible shock. The client in irreversible shock will not be alert.

Which clinical manifestation would be most indicative of complete arterial obstruction in the lower extremities?

coldness Explanation: Coldness is the assessment finding most consistent with complete arterial obstruction. Other expected findings would include paralysis and pallor.Aching pain, burning sensations, numbness, and tingling are all earlier signs of tissue hypoxia and ischemia and are associated with incomplete obstruction.

After evaluating a client for hypertension, a physician orders atenolol, 50 mg P.O. daily. Which therapeutic effect should atenolol have?

decreased cardiac output and decreased systolic and diastolic blood pressure Explanation: As a long-acting, selective beta1-adrenergic blocker, atenolol decreases cardiac output and systolic and diastolic blood pressure; however, like other beta-adrenergic blockers, it increases peripheral vascular resistance at rest and with exercise. Atenolol may cause bradycardia, not tachycardia.

Following cardiac bypass surgery, the client has been referred to a cardiac rehabilitation exercise program. The client has type 1 diabetes and has bilateral leg discomfort with walking. The client is exercising using a stationary bicycle. The nurse should evaluate the client's response to exercise by assessing the presence of which condition?

diabetic neuropathy Explanation: A common complication of diabetes is diabetic neuropathy. Diabetic neuropathy results from the metabolic and vascular factors related to hyperglycemia. Damage leads to sensory deficits and peripheral pain. Muscle atrophy can result from disuse, but it is not a direct consequence of diabetes. Raynaud's disease is associated with vasospasms in the hands and feet. Transient ischemic attacks involve the cerebrum.

A nurse is preparing to begin one-person cardiopulmonary resuscitation. The nurse should first

establish unresponsiveness. Explanation: The correct sequence begins with establishing unresponsiveness. The nurse should then call for help, assess the client for breathing while opening the airway, deliver two breaths, and check for a carotid pulse.

The nurse caring for a client on the cardiac unit notices that the client's cardiac monitor shows ventricular fibrillation. What is the priority action by the nurse?

immediate defibrillation Explanation: When ventricular fibrillation is verified, the first intervention is defibrillation. It is the only intervention that will terminate this lethal dysrhythmia. Digoxin will not help in this situation. An I.V. line will need to be established, but it is not the priority. A pacemaker may be needed, but not until the client is stabilized.

Which activity is least effective in preventing sensory deprivation during a client's stay in the cardiac care unit?

keeping the door closed to provide privacy Explanation: Keeping the client's door closed is likely to contribute to feelings of isolation and sensory deprivation. Such activities as watching television, visiting with a relative, and reading a newspaper help prevent sensory deprivation and yet do not require physical effort.

The nurse advises a client recovering from a myocardial infarction to decrease fat and sodium intake. Which foods should the nurse instruct the client to avoid? Select all that apply.

pepperoni pizza bacon cheese soft drinks Explanation: Foods high in sodium include cheese, processed meats such as pepperoni and bacon, and soft drinks. Bacon and cheese also have a high fat content.

A nurse just received a shift report for a group of clients on the telemetry unit. Which client should the nurse assess first?

the client admitted with first-degree atrioventricular (AV) block whose cardiac monitor now reveals type II second-degree AV block Explanation: The client whose cardiac rhythm now shows type II second-degree AV block should be assessed first. The client's rhythm has deteriorated from first-degree heart block to type II second-degree AV block and may continue to deteriorate into a lethal form of AV block (known as complete heart block). The client who underwent cardioversion 2 days ago has likely had the underlying reason for the sustained ventricular tachycardia corrected. The client with a history of heart failure may have chronic bibasilar crackles and pitting edema of both feet. Therefore, assessing this client first is not necessary. The client who underwent PCI with stenting was at risk for reperfusion arrhythmias and/or bleeding from the arterial puncture site but could be considered to be stable 24 hours post-procedure.

A client with severe angina pectoris and electrocardiogram changes is seen in the emergency department. What laboratory studies would the nurse most likely anticipate?

troponin Explanation: This client exhibits signs of myocardial infarction (MI), and the most accurate serum determinant of an MI is troponin level. Creatine kinase, lactate dehydrogenase, and myoglobin tests can show evidence of muscle injury, but they are less specific indicators of myocardial damage than troponin.

A registered nurse is mentoring a new graduate nurse. Which action by the new graduate demonstrates a need for further teaching?

turns the defibrillator to synchronize before defibrillating a client with ventricular fibrillation Explanation: The synchronizer switch should be turned "off" when defibrillating. All other answers are correct and do not require further teaching.

The nurse interprets the rhythm strip (see figure) from a client's bedside monitor as which rhythm?

ventricular tachycardia Explanation: This rhythm is ventricular tachycardia, which is characterized by an absent P wave and a heart rate of 140 to 220 bpm. Ventricular tachycardia requires immediate intervention, usually with lidocaine.

A nurse suspects that a client has digoxin toxicity. The nurse should assess for

vision changes. Explanation: Vision changes, such as halos around objects, are signs of digoxin toxicity. Hearing loss can be detected through hearing assessment; however, it isn't a common sign of digoxin toxicity. Intake and output aren't affected unless there is nephrotoxicity, which is uncommon. Gait changes are also uncommon.

A client returns to the nursing unit following successful synchronized cardioversion using transthoracic chest wall patches. What should the nurse assess when the client returns to the room? Select all that apply.

vital signs skin of chest wall level of consciousness cardiac rhythm Explanation: Vital signs give an important initial assessment of this client's status. The client may experience burns from the patches and current used for the cardioversion. Therefore, it is important to assess the skin of the chest wall for redness or burns. Because conscious sedation is used for this procedure, assessing the client's level of consciousness also is an important initial step. Attaching the client to cardiac monitoring is also important to assess rhythm abnormalities. There is no arterial puncture associated with the procedure.

A client had a repair of a thoracoabdominal aneurysm 2 days ago. Which finding should the nurse consider unexpected and report to the health care provider immediately?

weakness and numbness in the lower extremities Explanation: One of the complications of a thoracoabdominal aneurysm repair is spinal cord injury. Therefore, it is important for the nurse to assess for signs and symptoms of neurologic changes at and below the site where the aneurysm was repaired. The client is expected to have moderate pain following surgery. An elevated heart rate is expected after physical exertion. It is important to monitor urine output following aneurysm surgery, but a urine output of 2,000 mL in 24 hours is adequate following surgery.

Which statement would lead the nurse to determine that a client lacks understanding of the client's acute cardiac illness and the ability to make lifestyle changes?

"I already have my airline ticket, so I won't miss my meeting tomorrow." Explanation: Leaving the hospital and immediately flying to a meeting indicate poor judgment by the client and little understanding of what lifestyle changes the client needs to make. The other statements show that the client understands some of the changes that need to be made in order to decrease stress and lead a healthier lifestyle.

A nurse is performing discharge teaching with a client who has an implantable cardioverter defibrillator (ICD) placed. Which client statement indicates effective teaching?

"I'll keep a log of each time my ICD discharges." Explanation: The client stating that they should keep a log of all ICD discharges indicates effective teaching. This log helps the client and physician identify activities that may cause the arrhythmias that make the ICD discharge. The client should also record the events right before the discharge. Clients with ICDs should avoid contact sports such as football. They must also avoid magnetic fields, which could permanently damage the ICD. Household appliances don't interfere with the ICD.

A client with refractory angina pectoris is scheduled for a percutaneous transluminal coronary angioplasty (PTCA). The cardiologist orders an infusion of abciximab. Before beginning the infusion, the nurse should ensure the client has

A client with refractory angina pectoris is scheduled for a percutaneous transluminal coronary angioplasty (PTCA). The cardiologist orders an infusion of abciximab. Before beginning the infusion, the nurse should ensure the client has

The nurse on a medical unit is caring for a 61-year-old male client with multiple myeloma who presents with coughing, fever, and chills. The nurse is developing a plan of care for this client. Which potential nursing intervention(s) would the nurse include? Select all that apply.

Administer a broad-spectrum antibiotic. Teach pulmonary hygiene. Provide supplemental oxygen. Administer an antipyretic. Provide instruction to avoid infection.

A client in the emergency department reported vomiting and diarrhea for the previous 24 hours. The client's blood pressure is 90/60 mm Hg, respiration is 20 breaths per minute, heart rate is 92 beats per minute, and temperature is 37.5° C (99.5° F). Which intervention will the nurse perform first?

Assess for dehydration. Explanation: The priority for this client is assessing the problem. Then the nurse should treat the fluid volume deficit, then the temperature. This client has hypotension, and the nurse would raise the legs, not the head, of the bed first to improve perfusion to the brain, as it is the least restrictive intervention.

A nurse is caring for 4 clients on the cardiac unit. Which client has the greatest risk for contracting infective endocarditis?

a client 4 days postoperative after mitral valve replacement Explanation: Having prosthetic cardiac valves places the client at high risk for infective endocarditis. Hypertrophic cardiomyopathy and repaired ventricular septal defects are moderate risks for infective endocarditis. Coronary stent placement isn't a risk factor for infective endocarditis.

Which client admitted to the emergency department should the nurse see first?

a client experiencing a "ripping" sensation in the chest Explanation: A client experiencing a "ripping" sensation in the chest is indicative of a ruptured thoracic aneurysm and warrants an immediate intervention. While a blood pressure of 170/95 mm Hg is high, there is not enough information that suggests that this client is a higher priority than the others. A urine output of 240 mL in 12 hours is less than 30 ml/hour; however, this is this client's only problem now, and the nurse can investigate the cause next. A client experiencing bloody stools will need to be seen; however, no other information is present that would warrant this client being seen first.

The nurse is caring for a client prescribed IV heparin for treatment of thromboembolism. The client is prescribed 18 units/kg/hr. The client weighs 145 lb (66 kg). The heparin comes from the pharmacy as 25,000 units in 250 mL of D5W. How many mL/hr should this client receive? Round to the nearest whole number.

12 Explanation: The recommended dose of 18 units/kg should be obtained by multiplying the weight in kilograms by 18 units. 66 kg × 18 units = 1188 units/hr. Concentration for the medication is 25,000 units/250 mL. Use the formula Desired/Have × Volume: 1188 units/25,000 units × 250 mL = 11.88 mL/hr or 12 mL/hr.

The nurse teaches a client who has recently been diagnosed with hypertension about following a low-calorie, low-fat, low-sodium diet. Which menu selection would best meet the client's needs?

baked chicken, an apple, and a slice of white bread Explanation: Processed and cured meat products, such as cold cuts, ham, and hot dogs, are all high in both fat and sodium and should be avoided on a low-calorie, low-fat, low-salt diet. Dietary restrictions of all types are complex and difficult to implement with clients who are basically asymptomatic.

The nurse is assessing a client with an atrial septal defect (ASD). Which finding requires immediate nursing intervention?

client having an uneven smile and facial droop Explanation: A fixed S2 split is the hallmark of ASD. The neurologic finding of a facial droop could indicate embolization and stroke; the nurse should notify the healthcare provider immediately. If the client has missed a medication, the nurse should measure the vital signs and administer the medication as soon as possible; however, symptoms of stroke are the priority. The nurse should further assess tachycardia to determine the underlying cause, such as pain or fever, before intervening.

The nurse has completed an assessment on a client with a decreased cardiac output. Which findings should receive the highest priority?

confusion, urine output 15 mL over the last 2 hours, orthopnea Explanation: A low urine output and confusion are signs of decreased tissue perfusion. Orthopnea is a sign of left-sided heart failure. Crackles, edema, and weight gain should be monitored closely, but the levels are not as high a priority. With atrial fibrillation, there is a loss of atrial kick, but the blood pressure and heart rate are stable.

A client with heart failure will take oral furosemide at home. To help the client evaluate the effectiveness of furosemide therapy, the nurse should teach the client to:

weigh daily. Explanation: Monitoring daily weight will help determine the effectiveness of diuretic therapy. A client who gains weight without diet changes most probably is retaining fluids, so the diuretic therapy should be adjusted. Blood pressure monitoring is useful when diuretics are prescribed to control blood pressure. However, in clients with heart failure, the primary indication is to promote sodium and water excretion by the kidneys. While it may be useful to monitor intake and urinary output in the hospital, daily weights are a sensitive indicator of fluid status and more practical for home management. The client may be told to eat a potassium-rich diet; however, serum potassium levels are not used to determine the effectiveness of diuretic therapy

A client comes to the physician's office for a follow-up visit 4 weeks after suffering a myocardial infarction (MI). The nurse takes this opportunity to evaluate the client's knowledge of the ordered cardiac rehabilitation program. Which evaluation statement suggests that the client needs more instruction?

"Client walks 4 miles (6.4 kilometers) in 1 hour every day." Explanation: Four weeks after an MI, a client's walking program should aim for a goal of 2 miles (3.2 kilometers) in less than 1 hour. Walking 4 miles (6.4 kilometers) in 1 hour is excessive and may induce another MI by increasing the heart's oxygen demands. Therefore, this client requires appropriate exercise guidelines and precautions. Performing relaxation exercises; following a low-fat, low-cholesterol diet; and seeking emergency help if the heart rate increases markedly at rest indicate understanding of the cardiac rehabilitation program. For example, the client should reduce stress, which speeds the heart rate and thus increases myocardial oxygen demands. Reducing dietary fat and cholesterol intake helps lower risk of atherosclerosis. A sudden rise in the heart rate while at rest warrants emergency medical attention because it may signal a life-threatening arrhythmia and increase myocardial oxygen demands.

A client with angina pectoris must learn how to reduce risk factors that exacerbate this condition. When developing the client's care plan, which expected outcome should a nurse include?

"Client will verbalize the intention to stop smoking." Explanation: A client with angina pectoris should stop smoking at once because smoking increases the blood carboxyhemoglobin level; this increase, in turn, reduces the heart's oxygen supply and may induce angina. The client must seek immediate medical attention if chest pain doesn't subside after three nitroglycerin doses taken 5 minutes apart; serious myocardial damage or even sudden death may occur if chest pain persists for 2 hours. To improve coronary circulation and promote weight management, the client should get regular daily exercise. The client should eat plenty of fiber, which may decrease serum cholesterol and triglyceride levels and minimize hypertension, in turn reducing the risk for atherosclerosis (which plays a role in angina).

A nurse is teaching a client who will soon be discharged with a prescription for warfarin. Which statement should the nurse include in discharge teaching?

"Don't take aspirin while you're taking warfarin." Explanation: Because aspirin decreases platelet aggregation and interferes with clotting, concomitant use of aspirin with warfarin, an anticoagulant, may lead to excessive anticoagulant effects and bleeding. Warfarin therapy is most effective with consistent dietary intake of vitamin K. Increasing intake of foods rich in vitamin K, such as broccoli, could change the client's warfarin dose requirements. Although warfarin interrupts the normal clotting cycle, it doesn't dissolve clots that have already formed. The client should take warfarin exactly as ordered to maintain the desired level of anticoagulation. Doubling a dose could cause bleeding.

A client is admitted to the hospital for evaluation of recurrent episodes of ventricular tachycardia as observed on Holter monitoring. The client is scheduled for electrophysiology studies (EPS) the following morning. Which statement should the nurse include in a teaching plan for this client?

"During the procedure, the health care provider will insert a special wire used to increase the heart rate and produce the irregular beats that caused your signs and symptoms." Explanation: The purpose of EPS is to study the heart's electrical system. During this invasive procedure, a special wire is introduced into the heart to produce dysrhythmia. To prepare for this procedure, the client should be NPO for 6 to 8 hours before the test, and all antiarrhythmics are held for at least 24 hours before the test in order to study the dysrhythmia without the influence of medications. Because the client's verbal responses to the rhythm changes are extremely important, sedation is avoided if possible.

The nurse instructs a client on the use of transdermal nitroglycerin 0.2 mg/hour patch for angina pectoris. Which client statement indicates that teaching was effective?

"I should report any skin irritation to the healthcare provider." Explanation: Because transdermal nitroglycerin can cause skin irritation, this should be reported to the healthcare provider. The site to apply the patch should be rotated every day to prevent sensitization and tolerance. The medication pad should not be touched, because this could cause the drug to be absorbed through the fingers. The medication should be stored away from temperature and humidity extremes because this may inactivate the drug.

The nurse is caring for a client admitted for a quadruple coronary artery bypass graft. Which statements by the client indicate that preoperative teaching has not been effective? Select all that apply.

"I will be relieved to have this surgery over with; I have a very busy schedule at work right now." "I had stopped smoking a month before the surgery; however, I will be able to start again once I have recovered." Explanation: Both of these statements indicate that the client believes the surgery will solve the problem and lifestyle changes are not necessary. There is no demonstration of understanding of preoperative teaching. "I know that I will have to perform deep breathing and coughing exercises to prevent complications," "I will be on a heart monitor and a respirator to help me breathe," and "I understand that I need to change my eating habits and activity levels to keep my heart healthy" are all positive statements that indicate a good understanding of the teaching, indicating the client is an active participant and is following guidelines to help in recovery after the surgery and promote heart health.

A client weighs 300 lb (136 kg) and has a history of deep vein thrombosis and thrombophlebitis. When coaching a client about behaviors to maintain health, the nurse determines that the client has understood the nurse's instructions when the client makes which statement?

"I'll try to lose weight by following a reduced-calorie, balanced diet." Explanation: The client is at risk for development of varicose veins. Therefore, prevention is key in the treatment plan. Maintaining ideal body weight is the goal. In order to achieve this, the client should consume a balanced diet and participate in a regular exercise program. Performing leg lifts improves muscle strength, but it is more important for the client to increase exercise by walking. Wearing support stockings is helpful to promote circulation, but the client should not roll the stockings at the top to hold the stockings up as this would decrease circulation at the knees.

The nurse is teaching a client about actions to control manifestations of left-sided heart failure. Which statement by the client indicates appropriate understanding?

"If I have trouble breathing, I will sit in my recliner with my head up." Explanation: The decreased cardiac output that results from left-sided heart failure causes blood to accumulate in the pulmonary system. This produces pulmonary edema and difficulty breathing when lying flat. Weight gain is common with heart failure, but small fluctuations are normal with routine intake and output of food and fluids. Therefore clients are not expected to report a weight change unless it is more than 3 pounds (1.4 kg) in 48 hours. Nitroglycerin tablets are used to treat angina, not a rapid pulse. Elastic stockings may help control peripheral edema, but they do not help left-sided heart failure.

A client is participating in a cardiac rehab program after a myocardial infarction and, at this point, sexual activity is allowed to be resumed. Which of the following statements by the nurse is accurate regarding the resumption of sexual activity? Select all that apply.

"It is best to be well rested and in familiar surroundings." "A comfortable position should be maintained during intercourse." Explanation: Once a client has recovered from a myocardial infarction, sexual activity may be resumed. The client would be taught to be well rested and in familiar surroundings to decrease any additional anxiety surrounding the resumption. The client would also be taught to assume a comfortable position to prevent any additional strain on the heart. Anal activity would be a caution as the vagus nerve could be stimulated and result in dysrhythmias. Chest pain would not commonly occur if the client had recovered.

A client with a history of angina and intermittent claudication reports pain in both legs with a need to stop and rest after ambulating down the hall. Which statement by the nurse best addresses this concern??

"The pain is probably related to inadequately oxygenated blood getting through the arteries into the muscles of your legs." Explanation: When there is a history of atherosclerosis affecting the heart and resulting in intermittent claudication, there is arterial insufficiency. This results in inadequate provision of oxygenated blood to the muscles when there is an increase in muscle demand. This results in the pain of intermittent claudication. Thus, the nurse's response should explain there is inadequate oxygen reaching the leg muscles. The choices that mention "venous congestion" and "inadequate removal of carbon dioxide" refer to problems with venous congestion rather than arterial perfusion and do not directly address the reason for the pain. Stating that the pain is related to atherosclerosis does not explain why the client feels pain in the legs.

The recipient of a donated organ asks the nurse, "What did the donor die from?" Which response by the nurse is most appropriate?

"The transplant coordinator can give you information about the donor's medical history." Explanation: Confidentiality of the potential donor is always maintained unless the recipient and donor families both sign confidentiality waivers; however, medical history, such as history or hepatitis or HIV infection, is permitted. The transplant coordinator is the liaison for information regarding the donor.

A home care nurse is visiting a left-handed client who has an implantable cardioverter-defibrillator (ICD) implanted in their left chest. The client tells the nurse how excited they are because the client's planning to go rifle hunting with a grandson. How should the nurse respond?

"You can't shoot a rifle left-handed because the rifle's recoil will traumatize the ICD site." Explanation: The recoil from the rifle can damage the ICD, so the client should be warned against shooting a rifle with the left hand. Close proximity to a rifle won't cause the ICD to fire inadvertently. The client shouldn't take an extra dose of antiarrhythmic.

A client has a heart rate of 170 beats/minute. The physician diagnoses ventricular tachycardia and orders lidocaine hydrochloride, an initial I.V. bolus of 50 mg followed in 5 minutes by a second 50-mg bolus, then continuous I.V. infusion at 2 mg/minute. The nurse can expect the client to begin experiencing an antiarrhythmic effect within

1 to 2 minutes after I.V. bolus administration. Explanation: Lidocaine exerts its antiarrhythmic effect in 1 to 2 minutes after I.V. bolus administration. A continuous I.V. infusion will maintain lidocaine's antiarrhythmic effect for as long as the drip is used. Lidocaine provides antiarrhythmic effects for only 15 minutes after the I.V. infusion is stopped.

A client with deep vein thrombosis has an I.V. infusion of heparin infusing at 1,500 units/hour. The concentration in the bag is 25,000 units/500 ml. How many milliliters of solution should the nurse document as intake from this infusion for an 8-hour shift? Record your answer using a whole number.

240 Explanation: First, calculate how many units are in each milliliter of the medication. Set up the given ratio and reduce: 25,000 units / 500 ml = 50 units / 1 ml. Next, calculate how many milliliters the client receives each hour. Multiply the known ratios so that the units cancel out and you are left with ml/hr: 1 ml / 50 units × 1,500 units / 1 hr = 1,500/50 ml/hr = 30 ml/hr. Last, multiply by 8 hours: 30 ml/hour × 8 hours = 240 ml. There were 240 ml of solution administered over the shift.

Furosemide is administered intravenously to a client with heart failure. How soon after administration should the nurse begin to see evidence of the drug's desired effect?

5 to 10 minutes Explanation: After intravenous injection of furosemide, diuresis normally begins in about 5 minutes and reaches its peak within about 30 minutes. Medication effects last 2 to 4 hours. When furosemide is given intramuscularly or orally, drug action begins more slowly and lasts longer than when it is given intravenously.

A nurse is caring for a client with first-degree atrioventricular (AV) block. When instructing the spouse using a diagram, identify the area in the conduction cycle where this block occurs.

A nurse is caring for a client with first-degree atrioventricular (AV) block. When instructing the spouse using a diagram, identify the area in the conduction cycle where this block occurs.

The nurse is caring for a client with peripheral arterial occlusive disease (PAD). What nursing intervention is most appropriate to reduce platelet aggregation and promote circulation?

Administer clopidogrel. Explanation: Pharmacologic therapy for clients with PAD and claudication include pentoxifylline and cilostazal because these medications increase erythrocyte flexibility and decrease blood fibrinogen concentrations. Aspirin and clopidogrel are antiplatelet agents that prevent the formulation of emboli by reducing platelet aggregation. Statins are used to improve endothelial function. Therefore, clopidogrel should be administered because it is an antiplatelet agent that prevents the formulation of emboli by reducing platelet aggregation.

A client is started on digoxin. The health care provider (HCP) prescribes IV push doses of 0.5 mg now, 0.25 mg in 8 hr., and another 0.25 mg in another 8 hr. The client has a 1,000 mL bag of normal saline infusing at 25 mL/hr. What action should the nurse perform?

Administer each dose of medication over 5 minutes via IV push. Explanation: Digoxin is a potent cardiovascular drug that both slows conduction and increases contractility of the heart. Digoxin is administered slowly via IV push. Although each 1 mL can be diluted in 4 mL of SW, NS, D5W, or LR for injection, it is not added to the IV bag of solution or given over a 30-minute duration. There is no need to question the HCP's prescription at this time. Because digoxin is a new medication for this client and because it takes this type of dosing to reach a therapeutic level, dosing such as the one described is typical when the medication is first initiated. It is a type of loading dose protocol and for digoxin, sometimes referred to as digitalization.

A client admitted for a myocardial infarction (MI) develops cardiogenic shock. An arterial line is inserted. Which prescription from the health care provider should the nurse verify before implementing?

Administer metoprolol 5 mg IV push. Explanation: Metoprolol is indicated in the treatment of hemodynamically stable clients with an acute MI to reduce cardiovascular mortality. Cardiogenic shock causes severe hemodynamic instability and a beta blocker will further depress myocardial contractility. The metoprolol should be discontinued. The decrease in cardiac output will impair perfusion to the kidneys. Cardiac output, hemodynamic measurements, and appropriate interventions can be determined with a PA catheter. Dobutamine will improve contractility and increase the cardiac output that is depressed in cardiogenic shock.

A client is hospitalized following a report of dizziness, shortness of breath, and chest pain. Based on the ECG rhythm, the client is scheduled for a transesophageal echocardiogram (TEE) today. Which nursing intervention would be appropriate at this time?

Administer oxygen via nasal cannula as prescribed. Explanation: After analyzing the waveform, it is noted that the client is experiencing atrial fibrillation and is symptomatic; therefore, because of the client's symptoms, the nurse would administer oxygen. Initiating a medication such as a heparin requires a health care provider order. Deep breathing exercises assist to open airways and reestablish lung function following surgery. A TEE is sometimes prescribed before electrical cardioversion to ensure that there are no clots in the atria; if none are found, then the cardioversion can be safely performed.

The nurse is admitting an older adult to the hospital. The echocardiogram report revealed left ventricular enlargement. The nurse notes 2+ pitting edema in the ankles when getting the client into bed. Based on this finding, what should the nurse do first?

Assess respiratory status. Explanation: The ankle edema suggests fluid volume overload. The nurse should assess respiratory rate, lung sounds, and SpO2 to identify any signs of respiratory symptoms of heart failure requiring immediate attention. The nurse can then draw blood for laboratory studies, insert the Foley catheter, and weigh the client.

A nurse places electrodes on a collapsed individual who was visiting a hospitalized family member. The monitor exhibits the following. Which interventions would the nurse do first?

Assess the client's airway, breathing, and circulation. Explanation: The rhythm the client is experiencing is ventricular tachycardia (VT). Although all of the options listed are appropriate for someone with stable VT, it is not yet known whether the client's VT is stable, unstable, or pulseless. Therefore, the nurse must first assess the airway, breathing, circulation, and level of consciousness to establish the client's stability. Different actions are required if the client's VT is unstable or pulseless.

A client's electrocardiogram (EKG) tracing shows normal sinus rhythm followed by three premature ventricular contractions (PVCs) and a return to normal sinus rhythm. What is the priority action of the nurse?

Assess the client's apical-radial pulse rate. Explanation: Nonsustained ventricular tachycardia is several consecutive PVCs followed by the return to normal sinus rhythm. PVCs may reduce the CO and lead to angina and heart failure depending on frequency. Because PVCs in CAD or acute MI indicate ventricular irritability the nurse should first assess the client's physiologic response to PVCs by obtaining the client's apical-radial pulse rate, since PVCs often do not generate a sufficient ventricular contraction to result in a peripheral pulse. This can lead to a pulse deficit. Assessment of the client's hemodynamic status is important to determine if treatment with drug therapy is needed. Treatment relates to the cause of the PVCs such as oxygen therapy for hypoxia, electrolyte replacement, and drug therapy includes beta-adrenergic blockers, procainamide, or amiodarone.

The monitor technician informs the nurse that the client has started having premature ventricular contractions every other beat. What should the nurse do first?

Assess the client's orientation and vital signs. Explanation: The priority action is to assess the client and determine whether the rhythm is life threatening. More information, including vital signs, should be obtained and the nurse should notify the HCP. A bolus of lidocaine may be prescribed to treat this arrhythmia. This is not a code-type situation unless the client has been determined to be in a life-threatening situation.

A client in the emergency department has symptoms of anxiety, a "racing heart," and dyspnea. The cardiac monitor shows sinus tachycardia with a heart rate of 122. What is the appropriate action of the nurse?

Assess the client's vital signs and oxygen saturation. Explanation: Sinus tachycardia has multiple causes; further assessment is needed before determining the treatment. Administration of beta blockers or diazepam may not be indicated depending on the cause of the sinus tachycardia. A 12-lead EKG and troponin level might be appropriate following assessment of the client.

A client is admitted with a 6.5-cm thoracic aneurysm. The nurse records findings from the initial assessment in the client's chart, as shown. At 1030, the client has sharp mid-chest pain after having a bowel movement. What should the nurse do first?

Assess the client's vital signs. Explanation: The size of the thoracic aneurysm is rather large, so the nurse should anticipate rupture. A sudden incidence of pain may indicate leakage or rupture. The blood pressure and heart rate will provide useful information in assessing for hypovolemic shock. The nurse needs more data before initiating other interventions. After assessment of vital signs, neurologic status, and pain, the nurse can then contact the HCP.

A nurse is assessing a client's right lower leg, which is wrapped with an elastic bandage, and finds it to be cooler and paler than the left lower leg. What should the nurse do next?

Assess the distal pulses. Explanation: Signs and symptoms of impaired circulation include numbness and cool, pale skin. The next assessment should be to determine if the distal pulses are present and equal to the opposite extremity. Elevating the extremity would aid resolution of edema but will not help circulation. Lowering the head of the bed will not help. The healthcare provider should be notified of all assessment findings, including assessment of distal pulses.

The nurse is assessing a client after an above-the-knee amputation and notes that blood has saturated through the distal part of the dressing. What should the nurse do immediately?

Assess vital signs. Explanation: The client should be evaluated for hemodynamic stability and extent of bleeding prior to calling the HCP. Direct pressure can be used prior to applying a tourniquet if there is significant bleeding. To avoid flexion contractures, which can delay rehabilitation, elevation of the surgical limb is contraindicated.

The telemetry unit nurse is caring for a client who was just transferred from the coronary care unit (CCU). The client reports anxiety because of receiving less monitoring than in the CCU. How can the nurse allay the client's fears?

Assign the same nurse to the client when possible. Explanation: Assigning the same nurse when possible provides continuity of care and stability, thereby reducing the client's anxiety. An anxiolytic might be counterproductive and "overkill;" the client needs reassurance first. The client might have been the "most stable" choice in the event of an urgent need for a CCU bed. A room close to nurses' station would provide this client with a sense of security because the nurses are close by in the event of an emergency.

The nurse is evaluating arterial wave formation from an arterial line and notes a slow upstroke. What is the best action by the nurse?

Auscultate heart sounds. Explanation: A slow upstroke can be indicative of aortic stenosis. The nurse should auscultate heart sounds for signs and symptoms of aortic stenosis such as prolonged systolic ejection murmur and paradoxical splitting of S2 heart sound. Auscultating lung sounds will not provide information relevant to stenosis of the aorta. Assessment of the wrist for hyperextension would be appropriate if the client were exhibiting tingling or numbness in the fingers. Assessing capillary refill time would be appropriate if the client were exhibiting signs/symptoms of decreased perfusion to the hand.

A client is taking spironolactone. Which change in the diet should the nurse teach the client to make when taking this drug?

Avoid eating foods high in potassium. Explanation: Spironolactone is a potassium-sparing diuretic that causes excretion of sodium. When taking this drug, it is important that the client not eat foods high in potassium to avoid elevating serum potassium levels.The client does not need to restrict sodium intake as the drug promotes sodium excretion.Unless contraindicated, the client needs to maintain an adequate fluid intake; however, the client does not need to increase fluid intake to 3,000 mL/day.Spironolactone does not affect iron levels.

A client admitted to the telemetry unit with newly diagnosed atrial fibrillation has been started on warfarin. What should the nurse instruct the client to do when taking this medication? Select all that apply.

Avoid injury to prevent bruising. Be careful using a razor or fingernail clippers. Report any change in color of urine or stool. Explanation: Warfarin is an anticoagulant used in clients with atrial fibrillation to reduce the risk of stroke or systemic embolization and, therefore, will put the client at risk for bleeding. The nurse should instruct the client to watch for signs of bleeding and prevent bruising. While good oral hygiene remains important, the nurse would advise against vigorous flossing and irritating the gums as it may increase the risk of bleeding. Warfarin does not affect the heart rate.

A nurse is checking laboratory values on a client who has crackles in the lower lobes, 2+ pitting edema, and dyspnea with minimal exertion. Which laboratory value does the nurse expect to be abnormal?

B-type natriuretic peptide (BNP) Explanation: The client's symptoms suggest heart failure. BNP is a neurohormone that's released from the ventricles when the ventricles experience increased pressure and stretch, such as in heart failure. A BNP level greater than 51 pg/ml is commonly associated with mild heart failure. As the BNP level increases, the severity of heart failure increases. Potassium levels aren't affected by heart failure. CRP is an indicator of inflammation. It's used to help predict the risk of coronary artery disease. There is no indication that the client has an increased CRP. There is no indication that the client is experiencing bleeding abnormalities, such as those seen with an abnormal platelet count.

A client is diagnosed with thrombophlebitis. What nursing action would demonstrate the appropriate level of activity for this client?

Bed rest with the affected extremity elevated Explanation: Elevation of the affected leg facilitates blood flow by the force of gravity and also decreases venous pressure, which in turn relieves edema and pain. Other answers are incorrect based on appropriate level of activity needed to assist the diagnosis. Bed rest with normal activity is incorrect because pain is not always experienced with a thrombophlebitis.

A client in the intensive care unit has an arterial line that reads 58/30 mm Hg on the monitor. What is the nurse's first A client with Buerger's disease has established a goal to stop smoking. Which medication would be the most helpful in attaining this goal??

Bupropion. Explanation: Bupropion, a non-nicotine medication, is used to promote smoking cessation. All types of nicotine should be avoided to prevent vasoconstriction. Nitroglycerin, used for angina, and ibuprofen, an anti-inflammatory medication, have no role in smoking cessation.

A nurse is teaching a client how to take nitroglycerin to treat angina pectoris. What should the nurse include in the instructions?

Call emergency medical services immediately if chest pain does not subside within 15 minutes. Explanation: Nitroglycerine given for treatment of angina should be taken in 5 minute intervals for up to 3 doses. If the pain does not subside after three doses, the patient should seek emergency care immediately. Expected side effects of nitroglycerine include headache that should decrease over time, dizziness, and flushing. A dry cough is a side effect of an angiotensin converting enzyme (ACE) inhibitor.

A client who has undergone a mitral valve replacement has had a mediastinal chest tube inserted. The client has persistent bleeding from the sternal incision during the early postoperative period. What actions should the nurse take? Select all that apply.

Check the postoperative CBC, INR, PTT, and platelet levels. Confirm availability of blood products. Monitor the mediastinal chest tube drainage. Explanation: The hemoglobin and hematocrit should be assessed to evaluate blood loss. An elevated INR and PTT and decreased platelet count increase the risk for bleeding. The client may require blood products depending on lab values and severity of bleeding; therefore, availability of blood products should be confirmed by calling the blood bank. Close monitoring of blood loss from the mediastinal chest tubes should be done. Warfarin is an anticoagulant that will increase bleeding. Anticoagulation should be held at this time. Information is needed on the type of valve replacement. For a mechanical heart valve, the INR is kept at 2 to 3.5. Tissue valves do not require anticoagulation. Dopamine should not be initiated if the client is hypotensive from hypovolemia. Fluid volume assessment should always be done first. Volume replacement should be initiated in a hypovolemic client prior to starting an inotrope such as dopamine.

The nurse is reviewing the Nurse's Notes and Laboratory Results of 0930 to develop a plan of care for this client.

Complete the following sentence by choosing from the lists of options. The client is at highest risk for developing dysrhythmias due to the client's potassium level and serum digoxin level

The nurse in the emergency department (ED) is caring for a 73-year-old male client with a history of chronic obstructive pulmonary disease (COPD) who presents with severe shortness of breath and new-onset pedal edema. The nurse is developing a plan of care for this client.

Complete the following sentence by choosing from the lists of options. The client is at highest risk for developing hepatic injury as evidenced by right upper quadrant tenderness and alanine aminotransferase (ALT)

A client with sepsis has continued to deteriorate and has become unresponsive. The nurse has inserted an intravenous line and assisted with the initiation of mechanical ventilation. Which is the highest priority for the nurse at this time?

Confirm the placement of the endotracheal tube. Explanation: Confirming the placement of the endotracheal tube ensures oxygenation. Oxygen is essential for life, so this action takes priority. Intravenous fluid resuscitation is the next priority based on client unresponsiveness. Assessing for the source of the infection should be explored by assessing the surgical site, blood, urine, and sputum culture studies. Urine output should be monitored to assess perfusion and for potential acute renal failure.

A nurse is caring for a client with end-stage heart failure who is awaiting a heart transplant. The client tells the nurse that they think they are going to die before a donor heart is found. The client also tells the nurse that they have not been attending a church but wants to talk with a priest. What action should the nurse take?

Contact the clergy member who is assigned to the transplant team. Explanation: Each multidisciplinary transplant team has a clergy person assigned. The nurse should contact that person and request that the assigned clergy visit the client. It isn't appropriate for the nurse to ask their own priest to see the client. Telling the client that they have nothing to worry about because donors are typically found offers false reassurance. Telling the client that it doesn't matter if they attend a church invalidates the client's concern.

The nurse is evaluating a hemodynamically unstable client with an arterial line and notes that the client has tachycardia, cool and clammy skin, a pericardial friction rub, and the arterial waveform shows an inspiratory systolic pressure that is 15 mm Hg less than the expiratory systolic pressure. What is the priority intervention by the nurse?

Contact the health care provider. Explanation: The priority action is to contact the health care provider because these symptoms are indicative of cardiac tamponade. Assessing the heart sounds is not indicated at this time. The square wave test would be performed to check for accuracy of the arterial readings and optimal wave formation. There is no indication for performing a manual blood pressure at this time.

A nurse is caring for a client with type 2 diabetes who has had a myocardial infarction (MI) and is reporting nausea, vomiting, dyspnea, and substernal chest pain. Which is the priority intervention?

Control the pain and support breathing and oxygenation. Explanation: Support of breathing and ensuring adequate oxygenation are the two most important priorities. Reducing the substernal pain is also important because upset and anxiety will increase the demand for oxygen in the body. Controlling nausea, vomiting, and anxiety are all secondary in importance. Prevention of complications is important following initial stabilization and control of pain.

For each assessment finding, click to specify if the finding indicates digoxin toxicity or salicylate toxicity. Each finding may support more than one condition.

Digoxin Dizziness, abdominal Discomfort, loss of appetite, Yellow-green halos. Salicylate Dizziness, abdominal discomfort Explanation: This older adult with atrial fibrillation, hypertension, and heart failure has multiple assessment findings that require follow-up. A loss of appetite and abdominal pain in a client with atrial fibrillation and heart failure, who is taking several medications, requires follow-up.The client has orthostatic hypotension, a drop in systolic blood pressure of 20 mm Hg or more or a drop in diastolic blood pressure of 10 mm Hg or more when changing from a lying to a standing position. This finding requires follow-up and can occur with dehydration. The tenting and slow return of the skin to normal when pinched is a sign of dehydration and requires follow-up. The client's heart rate is within normal range and does not require follow-up. The oxygen saturation level of 96% is within normal limits and does not require follow-up. The client's electrocardiogram (ECG) shows normal sinus rhythm and does not require follow-up.

After extensive cardiac bypass surgery, a client returns to the intensive care unit on dobutamine, 5 mcg/kg/minute I.V. Which classification best describes dobutamine?

Direct-acting beta-active agent Explanation: Adrenergic agents are classified according to their method of action and the type of receptor on which they act. Direct-acting agents act on the sympathetically innervated organ or tissue, whereas indirect-acting agents trigger the release of a neurotransmitter, usually norepinephrine. Dual-acting agents combine direct and indirect actions. Adrenergic agents act on alpha, beta, and dopamine receptors. Dobutamine acts directly on beta receptors. Thus, the drug can be described as a direct-acting beta-active agent.

A client is receiving cilostazol for intermittent claudication. What should the nurse ask the client to determine the effectiveness of the drug?

Do you have less pain in the legs?" Explanation: Cilostazol improves blood flow, and the client should have improved circulation in the legs as evident by less pain. The client does not have nerve impairment and should be able to wiggle the toes. Urination is not improved by taking cilostazol. Dizziness is a side effect of the drug, not an intended outcome.

A nurse notes that the client's PR interval is .17 and the QRS complex is .10. What action should the nurse take next?

Document the findings. Explanation: These are normal findings. The nurse should document the findings. A 12-lead ECG would be ordered if the client needs further evaluation in the event of an abnormal finding. Administering nitroglycerin is a routine intervention and not related to the measured PR and QRS intervals. Oxygen administration is not indicated in the presence of normal findings.

A client with venous thrombus reports having pain in the legs. What should the nurse do first?

Elevate the foot of the bed. Explanation: Venous stasis can increase pain. Therefore, proper positioning in bed with the foot of the bed elevated or when sitting up in a chair can help promote venous drainage, reduce swelling, and reduce the amount of pain the client might experience. Placing a pillow under the knees causes flexion of the joint, resulting in a dependent position of the lower leg and causing a decrease in blood flow. Fluids are encouraged to maintain normal fluid and electrolyte balance but do little to relieve pain. Therapeutic massage to the legs is discouraged because of the danger of breaking up the clot.

A client with deep vein thrombosis suddenly develops dyspnea, tachypnea, and chest discomfort. What should the nurse do first?

Elevate the head of the bed 30 to 45 degrees. Explanation: Elevating the head of the bed facilitates breathing because the lungs are able to expand as the diaphragm descends. Coughing and deep breathing do not alleviate the symptoms of a pulmonary embolus, nor does lung auscultation. The HCP must be kept informed of changes in a client's status, but the priority in this case is alleviating the symptoms.

A client with heart failure has assessment findings of jugular vein distension (JVD) when lying flat in bed. What is the best nursing intervention?

Elevate the head of the bed to 30 to 45 degrees and reassess JVD. Explanation: Jugular vein distension should be measured when the head of the client's bed is at 30 to 45 degrees. The healthcare provider may or may not need to be notified, based on the assessment findings with the head of the bed elevated. Further assessment should be performed, but this further assessment does not include obtaining orthostatic blood pressure readings, since these readings do not affect JVD.

The client with peripheral artery disease reports both legs hurt when walking. What should the nurse instruct the client to do?

Enroll in a supervised exercise training program. Explanation: Decreased blood flow is a common characteristic of all PVD. When the demand for oxygen to the working muscles becomes greater than the supply, pain is the outcome. The nurse should suggest that the client enroll in a supervised exercise training program that will assist the client to gradually increase walking distances without pain. Not walking and resting will not increase blood flow to the legs. Support stockings may be prescribed, but the client should improve the capacity to walk and obtain exercise.

A client newly diagnosed with deep vein thrombosis (DVT) of the left lower left extremity is on bed rest. What should the nurse instruct the unlicensed assistive personnel (UAP) providing routine morning care for the client to do?

Ensure that the lower extremity is elevated. Explanation: DVT causes edema; therefore, the UAP should elevate the extremity to promote venous return. Dependent positioning is appropriate for a client with arterial insufficiency. Placing a pillow under the knee would position the foot in a low position, and pressure behind the knee may obstruct venous flow. Massaging the extremity could dislodge the thrombus.

Which of the following outcomes is desired when a client with arterial insufficiency has poor tissue perfusion in the extremities? Select all that apply.

Extremities warm to touch. Decreased muscle pain with activity. Explanation: The desired outcome for the client with poor circulation to the extremities is evidence of adequate blood flow to the area. The temperature of the involved extremity is an important indicator for a client with peripheral vascular disease. The temperature will indicate the degree to which the blood supply is getting to the extremity. Warmth indicates adequate blood flow. Pain is also an indicator of blood flow. Pain, such as muscle pain, suggests ischemia and lack of oxygen that results when the oxygen demand becomes greater than the supply. Thus, a decrease in muscle pain with activity would suggest improvement in blood flow to the area. Improved respiratory status and clear lungs are unrelated to the poor tissue perfusion. Although participation in self-care measures is always helpful, this outcome is not a result of establishing circulation to the extremities.

The nurse on a medical unit is caring for a 61-year-old male client with multiple myeloma who presents with coughing, fever, and chills. Which assessment finding(s) requires follow up? Select all that apply.

Fever Breath sounds Heart rate Respiratory rate Sputum production The client with multiple myeloma is at risk for recurrent infections as the rapidly growing plasma cells crowd and compress the white blood cells (WBCs). Treatment with chemotherapy also affects WBC production. The client is experiencing a fever, a sign of infection, which requires follow-up because the client has a low WBC count secondary to chemotherapy and the disease process. Wheezing and crackles in the left lower lobe of the lung are abnormal breath sounds that require follow-up. This client has tachycardia with a heart rate of 102 beats/min. The client may be tachycardic due to anemia that occurs with proliferation of plasma cells and increased metabolism secondary to fever. A respiratory rate of 26 breaths/min is above the normal range and reflects the increased work of breathing secondary to an increased metabolism from fever.The client has thick yellow sputum which, in conjunction with a fever, requires follow-up to determine if the client has a respiratory infection. The client is alert and oriented to time, place, and person, a normal finding that does not require follow-up. The client's dorsalis pedis and radial pulses are strong; therefore, no follow-up is required.

The nurse in the emergency department (ED) is caring for a 73-year-old male client with a history of chronic obstructive pulmonary disease (COPD) who presents with severe shortness of breath and new-onset pedal edema. The nurse has reviewed the provider's orders from 1430 and is implementing the plan of care. Select the nursing intervention from each of the following categories. Note: Each category must have at least 1 option selected.

Hydration: Monitor fluid intake and output Inject furosemide 40 mg IV over 1-2 minutes Oxygenation: Monitor oxygen saturation by pulse oximetry Auscultate breath sounds

A client arrives in the emergency department with an ischemic stroke. What should the nurse do before the client receives tissue plasminogen activator (t-PA)?

Identify the time of onset of the stroke. Explanation: Studies show that clients who receive recombinant t-PA treatment within 3 hours after the onset of a stroke have better outcomes. The time from the onset of a stroke to t-PA treatment is critical. A complete health assessment and history is not possible when a client is receiving emergency care. Upcoming surgical procedures may need to be delayed because of the administration of t-PA, which is a priority in the immediate treatment of the current stroke. While the nurse should identify which medications the client is taking, it is more important to know the time of the onset of the stroke to determine the course of action for administering t-PA.

The nurse in the emergency department (ED) is caring for a 73-year-old male client with a history of chronic obstructive pulmonary disease (COPD) who presents with severe shortness of breath and new-onset pedal edema. The nurse is comparing the Nurse's Notes and Laboratory Results of 1000 on day 1 with those of 0730 on day 2. For each assessment finding, click to specify if the finding indicates that the client's condition has improved, declined, or remained the same.

Improved: O2 sat , SOB, Pedal edema, Breath sounds, RR, LFT No changes: Ab distention, Pedal pulses

A client is to have sclerotherapy to treat varicose veins. What information about the procedure should the nurse include in the teaching plan for this client?

It causes the veins to fade and disappear. Explanation: Sclerotherapy involves injecting small- and medium-sized varicose veins with a solution that scars and closes those veins. In a few weeks, the veins should fade and disappear. This procedure does not require anesthesia and can be done in a health care provider's (HCP's) office. Varicose veins can reoccur regardless of the procedure. Bruising is more likely following vein stripping or catheter assisted procedures.

The nurse reviews the morning laboratory results from a client admitted with a deep vein thrombosis. The client is receiving intravenous heparin. Based on the client's current laboratory values, what should the nurse do?

Maintain the current rate of the heparin infusion. Explanation: An aPTT of 65 seconds is considered therapeutic with a control of 30. Therapeutic levels for heparin are 1.5 to 2.5 times the control, which would make therapeutic level between 45 seconds and 75 seconds. The nurse should continue the infusion at the current rate and continue to monitor the client. The liver enzymes (AST, ALT) are within normal range; it is not necessary to notify the HCP. The BUN and creatinine are within normal limits; the client does not need to increase fluid intake beyond 3,000 mL. The hemoglobin and hematocrit are within normal limits; it is not necessary to obtain frequent oxygen saturation levels.

An older adult had a myocardial infarction (MI) 4 days ago. At 0930, the client's blood pressure is 102/64 mm Hg. After reviewing the client's progress notes (see chart), what should the nurse do first?

Notify the health care provider (HCP). Explanation: All of the 1200 hour assessments are signs of decreased cardiac output and can be an ominous sign in a client who has recently experienced an MI; the nurse should notify the HCP of these changes. Cardiac output and blood pressure may continue to fall to dangerous levels, which can induce further coronary ischemia and extension of the infarct. While the client is currently hypotensive, giving a fluid challenge/bolus can precipitate increased workload on a damaged heart and extend the myocardial infarction. Exercise or walking for this client will increase both the heart rate and stroke volume, both of which will increase cardiac output, but the increased cardiac output will increase oxygen needs especially in the heart muscle and can induce further coronary ischemia and extension of the infarct. The client is hypotensive. Although the client has decreased urinary output, this is the body's response to a decreasing cardiac output, and it is not appropriate to administer furosemide.

Before discharge from the hospital after a myocardial infarction, the nurse teaches the client to exercise by gradually increasing the distance walked. What should the nurse teach the client to do to determine whether to increase or decrease the exercise level?

Obtain the heart rate for 1 minute at the end of the walk. Explanation: The client who is on a progressive exercise program at home after a myocardial infarction should be taught to monitor the heart rate and take the pulse at the end of each walk. The heart rate can be expected to increase with exercise, but the client should not increase the exercise if the heart rate increases more than about 25 bpm from baseline or exceeds 100 to 125 bpm. The nurse should also teach the client to discontinue exercise if chest pain occurs. The number of steps and the time to walk a mile are not determining factors for increasing the amount of exercise as long as the heart rate remains within range. The respirations may increase, but do not determine the ability to increase the exercise unless the client becomes short of breath.

The nurse should adjust a client's heparin dose according to a prescribed anticoagulation order based on maintaining which laboratory value at what therapeutic level for anticoagulant therapy?

Partial thromboplastin time, 1.5 to 2.5 times the normal control. Explanation: The nurse should adjust the heparin dose to maintain the client's partial thromboplastin time between 1.5 and 2.5 times the normal control. The prothrombin time and International Normalized Ratio are used to maintain therapeutic levels of warfarin, oral anticoagulation therapy. The thrombin clotting time is used to confirm disseminated intravascular coagulation.

Upon assessment of third-degree heart block on the monitor, what should the nurse do first?

Place transcutaneous pads on the client. Explanation: Transcutaneous pads should be placed on the client with third-degree heart block. For a client who is symptomatic, transcutaneous pacing is the treatment of choice. The hemodynamic stability and pulse should be assessed prior to calling a code or initiating CPR. Defibrillation is performed for ventricular fibrillation or ventricular tachycardia with no pulse.

A client suddenly develops paroxysmal supraventricular tachycardia (PSVT) at a rate of 180 bpm. Current vital signs: blood pressure 90/45 mm Hg, heart rate 180 bpm, respirations 30 breaths/min, O2 saturation 90% on room air. The client is diaphoretic and reports dizziness. What should the nurse do first?

Prepare defibrillator for synchronized cardioversion. Explanation: The nurse first should prepare the defibrillator for synchronized cardioversion. The client is experiencing PSVT with a heart rate of 180 bpm. PSVT with a heart rate of 180 bpm causes a decrease in cardiac output. The client's vital signs and symptoms reveal the client is becoming hemodynamically unstable and requires an immediate intervention. Atropine is not a treatment for PSVT. CPR is not indicated for PSVT. Caffeine use can contribute to PSVT, but the client requires defibrillation now.

A client has been admitted to the coronary care unit. The nurse observes third-degree heart block at a rate of 35 bpm on the client's cardiac monitor. The client has a blood pressure of 90/60 mm Hg. What should the nurse do first?

Prepare for transcutaneous pacing. Explanation: Transcutaneous pacemaker therapy provides an adequate heart rate to a client in an emergency situation. Defibrillation and a lidocaine infusion are not indicated for the treatment of third-degree heart block. Transcutaneous pacing is used temporarily until a transvenous or permanent pacemaker can be inserted.

A physician orders blood coagulation tests to evaluate a client's blood-clotting ability. The nurse knows that such tests are important in assessing clients at risk for thrombi, such as those with a history of atrial fibrillation, infective endocarditis, prosthetic heart valves, or myocardial infarction. Which test determines a client's response to oral anticoagulant drugs?

Prothrombin time (PT) Explanation: PT determines a client's response to oral anticoagulant therapy. This test measures the time required for a fibrin clot to form in a citrated plasma sample following addition of calcium ions and tissue thromboplastin and compares this time with the fibrin-clotting time in a control sample. The physician should adjust anticoagulant dosages as needed, to maintain PT at 1.5 to 2.5 times the control value. Bleeding time indicates how long it takes for a small puncture wound to stop bleeding. The platelet count reflects the number of circulating platelets in venous or arterial blood. PTT determines the effectiveness of heparin therapy and helps physicians evaluate bleeding tendencies. Physicians diagnose approximately 99% of bleeding disorders on the basis of PT and PTT values.

A client is admitted to the emergency department with sudden onset of chest pain. Which prescriptions should the nurse implement immediately? Select all that apply.

Provide oxygen. Administer nitroglycerin. Administer aspirin. Administer morphine. Explanation: When emergently managing chest pain, the nurse can use the memory mnemonic MONA to plan care: morphine, oxygen, nitroglycerin, and aspirin. A Foley catheter is not included in the emergent management of chest pain and can be inserted when the pain has been relieved and the client is stable. Acetaminophen is not used to manage chest pain.

Withhold the next scheduled dose. Explanation: Warfarin is an anticoagulant that prolongs the ability of blood to clot by inhibiting vitamin K. The international normalized ratio (INR) is a calculation that standardizes the prothrombin time (PT) to evaluate the effects of oral anticoagulant therapy. The normal range for INR is generally between 1 and 2. Therapeutic ranges are between 2 and 3 for most clients, and from 2.5 to 3.5 for clients with mechanical heart valves. Since this client's value is above the parameters for safe administration, the nurse should withhold the medication until the prescriber has been notified.

Reorient frequently to time, place and situation. Arrange for familiar pictures or special items at bedside. Spend time with the client, establishing a trusting relationship. Explanation: It is not unusual for the elderly client to become somewhat confused when "relocated" to the hospital, and this may be more difficult for those with known dementia. Frequent reorientation delivered patiently and calmly along with placing familiar items nearby so the client can see them may help decrease confusion related to hospitalization. Establishing a trusting relationship is important with every client but maybe more so with this client. Putting the client in a room further from the nursing station may decrease extra noise for the client, but will also make it more difficult to observe the client and maintain a safe environment. Procedures should be explained to the client prior to proceeding and should not be rushed. Visits by family and friends may help to keep the client oriented.

The nurse is observing the electrocardiogram (EKG) rhythm of a client with a permanent pacemaker and determines there is not a QRS complex that follows the pacemaker spike. Which follow-up action is most appropriate?

Report to the health care provider that the pacemaker is failing to capture. Explanation: Failure to capture is observed when the pacemaker fails to generate a complex; in this case the pacemaker fails to generate a ventricular complex with the QRS. This needs to be reported to the health care provider and the client should be assessed to determine any clinical manifestations of low blood flow due to this. The client's electrolyte levels would not have an effect on pacemaker function. Having the client take deep breaths and cough or lay on the right side with head of bed elevated would not have an effect on pacemaker function with failing to capture.

The nurse in the emergency department (ED) is caring for a 73-year-old male client with a history of chronic obstructive pulmonary disease (COPD) who presents with severe shortness of breath and new-onset pedal edema .The nurse is reviewing the Nurse's Notes and Laboratory Results of 1000 to develop a plan of care for this client. Which potential intervention(s) should the nurse include in the plan of care for this client?

Restrict sodium intake. Position for comfort. Administer supplemental oxygen. Administer a diuretic. Assess hydration status. Monitor oxygenation.

The nurse is evaluating a client who received tissue plasminogen activator (t-PA) following a myocardial infarction (MI). What is the expected outcome of this drug?

Revascularize the blocked coronary artery. Explanation: The thrombolytic agent t-PA, administered intravenously, lyses the clot blocking the coronary artery. The drug is most effective when administered within the first 6 hours after onset of MI. The drug does not reduce coronary artery vasospasm; nitrates are used to promote vasodilation. Arrhythmias are managed by antiarrhythmic drugs. Surgical approaches are used to open the coronary artery and re-establish a blood supply to the area.

The nurse is providing discharge teaching for a client with rheumatic endocarditis but no valvular dysfunction. On which nursing diagnosis should the nurse focus her teaching?

Risk for infection Explanation: Endocarditis is infection of the endocardium, heart valves, or a cardiac prosthesis, and clients are at high risk for relapse if they are not compliant with treatment or if they have invasive procedures. Therefore, clients with endocarditis have a Risk for infection. The nurse should stress to the client that they will need to continue antibiotics for a minimum of 5 years and that they will need to take prophylactic antibiotics before invasive procedures for life. There is no indication that the client has Chronic pain or Impaired memory. Because the client doesn't have valvular damage, Impaired gas exchange doesn't apply.

A client with end-stage heart failure is preparing for discharge. The client and their caregiver meet with the home care nurse and voice their concern that setting up a hospital bed in the bedroom will leave the client feeling isolated. Which suggestion by the home care nurse best addresses this concern?

Set up the hospital bed in the family room so the client can be part of household activities. Explanation: The client should be kept actively involved in the household to prevent feelings of isolation. This can be accomplished by setting up the hospital bed in the family room. Placing a chair in the bedroom allows the client periods of isolation when visitors aren't present. It's important for the client to have periods of rest; however, rest can be accomplished without keeping the client isolated in a bedroom. The needs of the client should be considered before the needs of the nurse who assesses the client during an occasional visit.

Which information about sublingual nitroglycerin tablets should the nurse include when instructing the client with angina?

Store the tablets in a tight, light-resistant container. Explanation: Clients should be instructed to keep nitroglycerin in a tightly closed, dark container and to replenish it frequently because it deteriorates rather rapidly.Nitroglycerin does not cause increased urine output.Clients should be instructed to use nitroglycerin at the first indication of chest pain and not to wait until pain becomes severe.

A nurse is caring for a client with advanced heart failure. The client can't care for themself and hasn't been able to eat for the past week because of dyspnea. The client doesn't want a feeding tube inserted and expresses their desire for "nature to take its course." The client's family is pleading with the client to have a feeding tube inserted. What is the most appropriate action for the nurse to take?

Talk with the client's family about the client's right to decide for themself. Explanation: Advocating for a client's wishes is a key nursing role. It's especially important when a client's family disagrees with the client's wishes. The nurse should be sure that the client has all the information needed to make an informed decision. Then the nurse should support the client's decision. The nurse shouldn't contact a clergyman without the client's consent, call a family conference, or schedule intubation in violation of the client's wishes.

The nurse is preparing a teaching plan for a client who is being discharged after being admitted for chest pain. The client has had one previous myocardial infarction 2 years ago and has been taking simvastatin 40 mg for the last 2 years. After reviewing the lab results for the client's cholesterol levels (see chart), what should the nurse do?

Tell the client that the cholesterol levels are within normal limits. Explanation: The serum cholesterol is within normal range for this client indicating the medication is effective. Since the cholesterol levels are within normal limits, it is likely that the client is taking the medication and asking may indicate the nurse has doubts or mistrusts that the client is taking the medication. The client does not need to change the diet at this point. Hemoglobin and hematocrit are not affected by simvastatin; since liver damage is a side effect of simvastatin, the nurse could review the liver function studies.

Which order(s) should the nurse anticipate in the care for this client? Select all that apply.

The nurse is collaborating with the health care provider on a plan of care for this client. Hold digoxin. Monitor serum digoxin levels. Monitor serum electrolytes. Provide continuous cardiac monitoring. Perform medication reconciliation. Provide education on medication safety. Explanation: This older adult with atrial fibrillation, hypertension, and heart failure has multiple assessment findings that require follow-up. A loss of appetite and abdominal pain in a client with atrial fibrillation and heart failure, who is taking several medications, requires follow-up.The client has orthostatic hypotension, a drop in systolic blood pressure of 20 mm Hg or more or a drop in diastolic blood pressure of 10 mm Hg or more when changing from a lying to a standing position. This finding requires follow-up and can occur with dehydration. The tenting and slow return of the skin to normal when pinched is a sign of dehydration and requires follow-up. The client's heart rate is within normal range and does not require follow-up. The oxygen saturation level of 96% is within normal limits and does not require follow-up. The client's electrocardiogram (ECG) shows normal sinus rhythm and does not require follow-up.

A nurse is caring for a client returning from cardiac catheterization. The nurse helps transfer the client back to bed. Which transfer technique uses appropriate ergonomic principles?

The nurse raises the bed for transfer, maintains a wide base of support during transfer, and lowers the bed before leaving the room. Explanation: The nurse should raise the bed for transfer, maintain a wide base of support during transfer, and lower the bed before leaving the room. Lowering the bed for a transfer places the nurse at risk for injury. Raising the bed before leaving the room places the client at risk for injury. The nurse should maintain a wide base of support for transfers and shouldn't encourage the client to grab or hold onto staff members during transfers. Although the nurse should explain the procedure to the client, the nurse shouldn't grab the client under the arms. This action could cause shoulder injury or nerve damage. The nurse shouldn't pull a client during transfers; doing so places the client at risk for skin-shear injuries.

A client with peripheral artery disease has chronic, severe bilateral pretibial and ankle edema the client is on complete bed rest. To maintain skin integrity, what should the nurse do?

Turn the client every 1 to 2 hours. Explanation: The client is at greater risk for skin breakdown in the lower extremities related to the edema and to remaining in one position, which increases capillary pressure. Turning the client every 1 to 2 hours promotes vasodilation and prevents vascular compression. Administering pain medication will not have an effect on skin integrity. Encouraging fluids is not a direct intervention for maintaining skin integrity, although being well hydrated is a goal for most clients. Maintaining hygiene does influence skin integrity but is secondary in this situation.

A client is scheduled for a cardiac catheterization. The nurse should do which preprocedural tasks? Select all that apply.

Verify the client has stopped taking anticoagulants if instructed by the health care provider. Check for iodine sensitivity. Verify that written consent has been obtained. Withhold food and oral fluids before the procedure. Explanation: For clients scheduled for a cardiac catheterization, it is important to assess for iodine sensitivity, verify written consent, and instruct the client to take nothing by mouth for 6 to 18 hours before the procedure. If the client is taking anticoagulant drugs, the nurse should ask the client if the health care provider has given instructions to withhold these medications. Oral medications are withheld unless specifically prescribed. A urinary drainage catheter is rarely required for this procedure.

The nurse in the emergency department (ED) is caring for a 73-year-old male client with a history of chronic obstructive pulmonary disease (COPD) who presents with severe shortness of breath and new-onset pedal edema. The nurse is reviewing the Nurse's Notes and Laboratory Results of 1000, on day 1 of admission, for this client. Which assessment finding(s) requires immediate follow-up by the nurse? Select all that apply.

Weight gain Breath sounds Liver function tests Hematocrit Abdominal assessment Explanation: This client with chronic obstructive pulmonary disease (COPD) presents with signs and symptoms consistent with right-sided heart failure. A weight gain of 8 lb. (3.6 kg) in 1 week is a significant increase that requires follow-up because it is a sign of fluid retention. Breath sounds of diffuse wheezing and crackles on lung auscultation suggest the buildup of fluid in the lungs due to right-sided heart failure. The client's liver function tests (ALT and AST) may be elevated due to hepatic congestion caused by right-sided heart failure and require follow-up.The client's hematocrit is elevated, most likely due to the chronic hypoxia that develops in COPD. This elevated level requires follow-up. A distended abdomen and right upper quadrant (RUQ) tenderness require follow-up because they are signs of hepatic injury that can occur with right-sided heart failure. The client is alert and oriented to person, place, time, and event, a normal finding that does not require follow-up.The client's bowel sounds are normal and do not require follow-up.

A client is receiving digoxin, and the pulse range is normally 70 to 76 bpm. After assessing the apical pulse for 1 minute and finding it to be 60 bpm, the nurse should do what first?

Withhold the digoxin. Explanation: The nurse's initial response should be to withhold the digoxin. The nurse should then notify the HCP if the apical pulse is 60 bpm or lower because of the risk of digoxin toxicity. The charge nurse does not need to be notified, but the nurse needs to document the notification and follow-up in the medical record.

A client is being treated for deep vein thrombosis (DVT) in the left femoral artery. The health care provider (HCP) has prescribed 60 mg of enoxaparin subcutaneously. Before administering the drug, the nurse checks the client's laboratory results. (See image.) Based on these results, what should the nurse do?

Withhold the dose of the medication and contact the HCP. Explanation: Based on the laboratory findings, prothrombin time and INR are at acceptable anticoagulation levels for the treatment of DVT. However, the platelets are below the acceptable level. Clients taking enoxaparin are at risk for thrombocytopenia. Because of the low platelet level, the nurse should withhold the enoxaparin and contact the HCP. The HCP, not the pharmacist, will make the decision about the dose of the enoxaparin. The decision about administering the drug will be based on laboratory results, not evidence of bruising or bleeding.

A client is about to undergo cardiac catheterization for which informed consent was obtained. As the nurse enters the room to administer sedation for the procedure, the client states, "I'm really worried about having this open heart surgery." Based on this statement, how should the nurse proceed?

Withhold the medication and notify the physician immediately. Explanation: The nurse should withhold the medication and notify the physician that the client does not understand the procedure. The physician then has the obligation to explain the procedure better to the client and determine whether or not the client understands. If the client does not understand, there cannot be a true informed consent. If the medication is administered before the physician explains the procedure, the sedation may interfere with the client's ability to clearly understand the procedure. The nurse may not just medicate the client and document the finding; the physician must be notified. The procedure does not need to be cancelled, only postponed until the client receives more education and is able to give informed consent.

A client with atrial fibrillation has been receiving warfarin. The INR is 4.5. What is the next action the nurse should take?

Withhold the next scheduled dose. Explanation: Warfarin is an anticoagulant that prolongs the ability of blood to clot by inhibiting vitamin K. The international normalized ratio (INR) is a calculation that standardizes the prothrombin time (PT) to evaluate the effects of oral anticoagulant therapy. The normal range for INR is generally between 1 and 2. Therapeutic ranges are between 2 and 3 for most clients, and from 2.5 to 3.5 for clients with mechanical heart valves. Since this client's value is above the parameters for safe administration, the nurse should withhold the medication until the prescriber has been notified.

The nurse is caring for a client with peripheral artery disease (PAD) who has just returned from having a percutaneous transluminal balloon angioplasty. Which finding requires immediate attention from the nurse?

a change in the intensity of the pulse from the baseline Explanation: A change in the intensity of a pulse maybe indicative of arterial closure and warrants immediate attention; the nurse should notify the health care provider (HCP) immediately. A pain level of 2 out of 10 it is not uncommon from the catheter insertion site especially after the placement of a stent. Shiny and hairless skin is expected in clients with PAD. A client undergoing a catheterization may experience pain at the catheterization site as large bore sheaths are place in the femoral artery. Because people with PAD have poor circulation in their lower extremities, it is possible for them to develop leg ulcers. However it is unlikely that the percutaneous transluminal balloon angioplasty caused this.

Which symptom should the nurse teach the client with unstable angina to report immediately to the health care provider (HCP)?

a change in the pattern of the chest pain Explanation: The client should report a change in the pattern of chest pain. It may indicate increasing severity of coronary artery disease. Pain occurring during stress or sexual activity would not be unexpected, and the client may be instructed to take nitroglycerin to prevent this pain. Pain during or after an activity such as lawn mowing also would not be unexpected; the client may be instructed to take nitroglycerin to prevent this pain or may be restricted from doing such activities.

The nurse is caring for a group of clients. Which client should the nurse see first?

a client with a placement of a coronary artery stent 30 minutes ago Explanation: The client who has just returned from having a stent placed in a coronary artery should be seen first. The nurse should assess this client to establish a baseline. Risks associated with a stent placement include a reocclusion, cardiac tamponade, dysrhythmias, bleeding, and thrombosis. While a new onset of atrial fibrillation is a concern, this client's heart rate is less than 100 bpm and is not showing signs of being hemodynamically unstable. A client with a history of sinus rhythm who will receive a beta-blocker is not a higher priority. While a client with stable angina who took a sublingual nitroglycerine 30 minutes ago will need to be assessed frequently, there is no evidence to suggest this client is currently experiencing chest pain.

While auscultating the heart sounds of a client with heart failure, the nurse hears an extra heart sound immediately after the second heart sound (S2). The nurse should document this as

a third heart sound (S3). Explanation: An S3 is heard following an S2, which commonly occurs in clients experiencing heart failure and results from increased filling pressures. An S1 is a normal heart sound made by the closing of the mitral and tricuspid valves. An S4 is heard before an S1 and is caused by resistance to ventricular filling. A murmur is heard when there is turbulent blood flow across the valves.

The client is admitted in septic shock. Which assessment data warrants immediate intervention by the nurse?

a urinary output of 50 mL in the past 3 hours Explanation: Sepsis can cause the release of myoglobin from the cells which will directly block the renal tubules, causing decreased urinary output. If it is not treated with hydration and antibiotics, the client could develop renal failure. A high white blood cell count is expected with sepsis. Temperature can be elevated or below normal in clients with sepsis. The elevated pulse and respirations are normal in the presence of infection and should be monitored. The saturated oxygen level is within normal limits as is the blood pressure.

A client with left-sided heart failure complains of increasing shortness of breath and is agitated and coughing up pink-tinged, foamy sputum. The nurse should recognize these findings as signs and symptoms of

acute pulmonary edema. Explanation: Shortness of breath, agitation, and pink-tinged, foamy sputum signal acute pulmonary edema. This condition results when decreased contractility and increased fluid volume and pressure in clients with heart failure drive fluid from the pulmonary capillary beds into the alveoli. In right-sided heart failure, the client would exhibit hepatomegaly, jugular vein distention, and peripheral edema. In pneumonia, the client would have a temperature spike and sputum that varies in color. Cardiogenic shock is indicated by signs of hypotension and tachycardia.

When assessing a client with left-sided heart failure, the nurse expects to note

air hunger. Explanation: With left-sided heart failure, the client typically has air hunger and other signs of pulmonary congestion. Ascites, jugular vein distention, and pitting edema of the legs are signs of right-sided heart failure.

The nurse is assessing a client who had an abdominal aortic aneurysm repair 2 hours ago. Which finding warrants further evaluation?

an arterial blood pressure of 80/50 mm Hg Explanation: A blood pressure of 80/50 mm Hg in a client who has just had surgical repair of an abdominal aortic aneurysm warrants further evaluation as this indicates decreased perfusion to the brain, heart, and kidneys. A BUN of 26 and a creatinine of 1.2 are normal findings. While +1 pedal pulses may be an abnormal finding, it is not uncommon, and it is important to compare this finding to previous assessments and note if this is a change of the strength of the pedal pulses. Absent bowel sound and mild abdominal distension is expected for a client immediately following surgery. However this finding should be monitored as it could indicate a paralytic ileus.

A client develops atrial fibrillation following an acute myocardial infarction. The physician orders digoxin, 0.125 mg I.M. daily. The nurse clarifies the order with the physician because I.M. administration of digoxin leads to

an increased serum creatine kinase (CK) level. Explanation: I.M. administration of digoxin isn't recommended because it causes severe pain at the injection site and increases serum CK, which complicates interpretation of enzyme levels. Regardless of the route of administration, digoxin doesn't increase the serum creatinine level. When digoxin is administered, the serum digoxin level will rise from zero, not decrease.

A client has a blockage in the proximal portion of a coronary artery. After learning about treatment options, the client decides to undergo percutaneous transluminal coronary angioplasty (PTCA). During this procedure, the nurse expects to administer an

anticoagulant. Explanation: During PTCA, the client receives heparin, an anticoagulant, as well as calcium agonists, nitrates, or both, to reduce coronary artery spasm. Nurses don't routinely give antibiotics during this procedure; however, because the procedure is invasive, the client may receive prophylactic antibiotics to reduce the risk of infection. An antihypertensive may cause hypotension, which should be avoided during the procedure. An anticonvulsant isn't indicated because this procedure doesn't increase the risk of seizures.

Three days after surgery to insert a mechanical mitral valve, the client asks what can be done to muffle the clicking sound since it is embarrassing and others will know an artificial valve is in the heart. The nurse's response should reflect the understanding that the client may be experiencing which concern?

anxiety related to altered body image Explanation: Verbalized concerns from this client may stem from anxiety over the insertion of a mechanical heart valve that makes an audible clicking sound and fear that others will become aware of the sound and ask questions. Although the client may experience anxiety related to the altered health status or may have a lack of knowledge regarding the postoperative course, the client is pointing out the changes in body image. The client is not concerned about altered tissue perfusion.

A client admitted with a massive myocardial infarction rapidly develops cardiogenic shock. Ideally, the physician would use the intra-aortic balloon pump (IABP) to support the injured myocardium. However, this client has a history of unstable angina pectoris, aortic insufficiency, hypertension, and diabetes mellitus. Which condition is a contraindication for IABP use?

aortic insufficiency Explanation: A history of aortic insufficiency contraindicates use of the IABP. Other contraindications for this therapy include aortic aneurysm, central or peripheral atherosclerosis, chronic end-stage heart disease, multisystemic failure, chronic debilitating disease, bleeding disorders, and a history of emboli. Unstable angina pectoris that doesn't respond to drug therapy is an indication for IABP, not a contraindication. Hypertension and diabetes mellitus aren't contraindications for IABP.

A client with chronic arterial occlusive disease undergoes percutaneous transluminal coronary angioplasty (PTCA) for mechanical dilation of the right femoral artery. After the procedure, the client will require long-term administration of

aspirin or clopidogrel. Explanation: After PTCA, the client begins long-term aspirin or clopidogrel therapy to prevent thromboembolism. Physicians order heparin for anticoagulation during this procedure; some physicians discharge clients with a prescription for long-term warfarin or low-molecular-weight heparin therapy. Pentoxifylline, a vasodilator used to treat chronic arterial occlusion, isn't required after PTCA because the procedure itself opens the vessel. The physician may order short-term acetaminophen therapy to manage fever or discomfort, but prolonged therapy isn't warranted. The client may need an antibiotic, such as penicillin or erythromycin, for a brief period to prevent infection associated with an invasive procedure; long-term therapy isn't necessary.

A client is admitted to the hospital through the emergency department with chest pain. Which intervention is the priority?

assessing troponin 1 levels Explanation: Troponin 1 rises with myocardial infarction. This assessment will best determine the cause of the client's chest pain and allow for immediate treatment. Monitoring the white blood count and platelet count and assessing the B-type natriuretic peptide levels are important, but not the priority.

Which condition most commonly results in coronary artery disease (CAD)?

atherosclerosis Explanation: Atherosclerosis (plaque formation), is the leading cause of CAD. Diabetes mellitus is a risk factor for CAD, but it isn't the most common cause. Myocardial infarction is a common result of CAD. Renal failure doesn't cause CAD, but the two conditions are related.

The client has had a myocardial infarction, and the nurse has instructed the client to prevent Valsalva's maneuver. The nurse determines the client is following the instructions when the client:

avoids holding the breath during activity. Explanation: Valsalva's maneuver, or bearing down against a closed glottis, can best be prevented by instructing the client to exhale during activities such as having a bowel movement or moving around in bed.Valsalva's maneuver is not prevented by having the client assume a side-lying position.Clenching the teeth will likely contribute to Valsalva's maneuver, not inhibit it.Drinking fluids through a straw has no effect on preventing or causing Valsalva's maneuver.

Which food should the nurse teach a client with heart failure to limit when following a 2-gram sodium diet?

canned tomato juice Explanation: Canned foods and juices such as tomato juice are typically high in sodium and should be avoided in a sodium-restricted diet. Canned foods and juices in which sodium has been removed or limited are available. The nurse should teach the client to read labels carefully. Apples and whole wheat breads are not high in sodium. Hamburger would have less sodium than canned foods or tomato juice.

A nurse is caring for a client with a history of cardiac disease and type 2 diabetes. The nurse is closely monitoring the client's blood glucose level. Which medication is the client most likely taking?

carvedilol Explanation: The nurse must monitor blood glucose levels closely in clients with type 2 diabetes who are taking beta-adrenergic blockers such as carvedilol, because beta-adrenergic blockers may mask the signs of hypoglycemia. The nurse should monitor QRS duration in clients taking procainamide and pulmonary function in clients taking amiodarone (because the drug may cause pulmonary fibrosis). Diltiazem may cause an increased PR interval or bradycardia.

A client reports recent onset of chest pain that occurs sporadically with exertion. The client also has fatigue and mild ankle swelling, which is most pronounced at the end of the day. The nurse suspects a cardiovascular disorder. What other client presentation increases the likelihood of a cardiovascular disorder?

clubbing of fingers Explanation: Common signs and symptoms of cardiovascular dysfunction include shortness of breath, chest pain, palpitations, fainting, fatigue, and peripheral edema. Clubbing of fingers indicates chronic hypoxemia, possibly as a result of undiagnosed heart disease. Urinary frequency is a symptom of a UTI or other urinary tract issue. Although irritability may occur if cardiovascular dysfunction leads to cerebral oxygen deprivation, this symptom more commonly reflects a respiratory or neurologic dysfunction. Lower substernal abdominal pain occurs with some GI disorders.

A client with a ventricular dysrhythmia is receiving intravenous lidocaine. For which assessment finding should the nurse suspect the client is experiencing toxicity from the medication?

confusion and restlessness Explanation: Confusion and restlessness are signs of lidocaine toxicity. Nausea and vomiting may occur with oral administration of mexiletine or tocainide — other class IB drugs. Pupillary changes and hypertension aren't signs of lidocaine toxicity, although visual changes and hypotension may occur as adverse reactions to class IB drugs.

The nurse is assessing a client with a known history of chronic heart failure. Which finding indicates poor perfusion to the tissues?

cool, pale extremities Explanation: In heart failure, the heart is unable to adequately meet the body's metabolic demands; in an attempt to supply major organs, less blood is circulated to extremities, leaving them cool, pale and potentially cyanotic. A blood pressure of 102/64 mm Hg is lower than average, but it may be normal for this client and would not indicate poor perfusion to tissues. It is not unusual for the client with heart failure to have a slightly elevated heart rate (unless taking medications to lower the heart rate) because the increased rate may help compensate for reduced stroke volume (and therefore, decreased cardiac output). Shortness of breath may occur with heart failure as a result of poor pumping action of the heart that allows fluid to accumulate in the lungs, however, it is not an indicator of peripheral perfusion.

A client is admitted for treatment of Prinzmetal's angina. When developing this client's care plan, the nurse should keep in mind that this type of angina can result from:

coronary artery spasm. Explanation: Prinzmetal's angina results from coronary artery spasm. Activities that increase myocardial oxygen demand may trigger angina of effort. An unpredictable amount of activity may precipitate unstable angina. Worsening angina is brought on by the same type or level of activity that caused previous angina episodes; anginal pain becomes increasingly severe.

The nurse is assessing a client with peripheral arterial disease who had a femoral-popliteal bypass. Which finding indicates improved arterial blood supply to the lower extremity?

decrease in muscle pain when walking Explanation: With increased blood supply to the leg there should be less or absent claudication (cramping pain in leg with walking). Pulses should be palpable with improved blood supply. Edema is associated with venous disease. Pallor with elevation and dependent rubor are symptoms of peripheral arterial disease.

A client with hypertrophic cardiomyopathy (HCM) is experiencing dyspnea, chest pain, syncope, fatigue, and palpitations and has an apical systolic thrill and heave, fourth heart sound (S4), and systolic murmur. Which nursing diagnosis should the nurse use to guide this client's care?

decreased cardiac output Explanation: Decreased cardiac output is an appropriate nursing diagnosis for a client with HCM because the hypertrophied cardiac muscle decreases the effectiveness of the heart's contraction, decreasing cardiac output. Heart failure may complicate HCM, causing fluid volume excess; therefore, the nursing diagnosis of risk for deficient fluid volume is not applicable. Ineffective peripheral tissue perfusion would be applicable if the client is experiencing an alteration in peripheral pulses, capillary refill time greater than 3 seconds, or a change in skin characteristics. Although it might seem that the diagnosis of risk for activity intolerance would be applicable because of dyspnea and fatigue, addressing cardiac output will help reduce these symptoms.

The nurse is caring for a client in the intensive care unit. Which drug is most commonly used to treat cardiogenic shock?

dopamine Explanation: Cardiogenic shock is when the heart has been significantly damaged and is unable to supply enough blood to the organs of the body. Dopamine, a sympathomimetic drug, improves myocardial contractility and blood flow through vital organs by increasing perfusion pressure. Enalapril is an angiotensin-converting enzyme inhibitor that directly lowers blood pressure. Furosemide is a diuretic and does not have a direct effect on contractility or tissue perfusion. Metoprolol is a adrenergic blocker that slows heart rate and lowers blood pressure; neither is a desired effect in the treatment of cardiogenic shock.

A visitor to the hospital has a cardiac arrest. When determining to use an automated external defibrillator (AED), the nurse should consider that AEDs are used in cardiac arrest in which circumstances?

early defibrillation in cases of ventricular fibrillation Explanation: AEDs are used for early defibrillation in cases of ventricular fibrillation. The American Heart Association and Canadian Heart and Stroke Foundation place major emphasis on early defibrillation for ventricular fibrillation and use of the AED as a tool to increase sudden cardiac arrest survival rates.

A client is taking spironolactone to control hypertension. The client's serum potassium level is [6 mEq/L (56mmol/L)]. For this client, the nurse's priority should be to assess their

electrocardiogram (ECG) results. Explanation: Although changes in all these findings are seen in hyperkalemia, ECG results should take priority because changes can indicate potentially lethal arrhythmias such as ventricular fibrillation. It wouldn't be appropriate to assess the client's neuromuscular function, bowel sounds, or respiratory rate for effects of hyperkalemia.

Following a myocardial infarction, a client develops an arrhythmia and requires a continuous infusion of lidocaine. To monitor the effectiveness of the intervention, the nurse should focus primarily on the client's:

electrocardiogram (ECG). Explanation: Lidocaine is an antiarrhythmic and is given for the treatment of cardiac irritability and ventricular arrhythmias. The best indicator of its effectiveness is a reduction in or disappearance of ventricular arrhythmias as seen on an ECG. Lidocaine level will be monitored but it is not the primary focus; troponin level monitors myocardial damage. Blood pressure, which can drop on lidocaine, does need to be monitored but the focus should be the ECG to evaluate the effectiveness of the medication.

The nurse observes that an older female has small to moderate, distended and tortuous veins running along the inner aspect of her lower legs. The nurse should:

encourage the client to avoid standing in one position for long periods of time. Explanation: The client has varicose veins, which are evident by the tortuous, distended veins where blood has pooled. To prevent pooling of the blood, the client should not stand in one place for long periods of time.It is not necessary to use compression devices, but the client could wear support hose if she stands for long periods of time.The client can consider cosmetic surgery to remove the distended veins, but there is no indication that the client should contact the health care provider at this point in time.The nurse can inspect the client's feet, but the client is not at risk for ulcers at this time.

A fourth heart sound (S4) indicates a

failure of the ventricle to eject all blood during systole. Explanation: An S4 occurs as a result of increased resistance to ventricular filling following atrial contraction. This increased resistance is related to decreased ventricular compliance. A dilated aorta doesn't cause an extra heart sound, though it does cause a murmur. A nurse hears decreased myocardial contractility as a third heart sound. A nurse doesn't hear an S4 in a normally functioning heart.

A client has risk factors for coronary artery disease, including smoking cigarettes, eating a diet high in saturated fat, and leading a sedentary lifestyle. Which coaching strategies from the nurse will be most effective in assisting the client improve his or her health?

helping the client establish a wellness vision to reduce the health risks Explanation: In health coaching, unlike traditional client education techniques in which the nurse provides information, the goal of coaching is to encourage the client to explore the reasons for the behavior and establish a vision for health behavior and the way he or she can make changes to improve their health behavior and reduce or eliminate health risks. When coaching a client, the nurse does not provide information, withhold praise, or instill fear.

In presenting a workshop on parameters of cardiac function, which conditions should a nurse list as those most likely to lead to a decrease in preload?

hemorrhage, sepsis, and anaphylaxis Explanation: Preload is the volume in the left ventricle at the end of diastole. It's also referred to as end-diastolic volume. Preload is reduced by any condition that reduces circulating volume, such as hemorrhage, sepsis, and anaphylaxis. Hemorrhage reduces circulating volume by loss of volume from the intravascular space. Sepsis and anaphylaxis reduce circulating volume by increased capillary permeability. Diuresis, vasodilation, and third spacing also reduce preload. Preload increases with fluid overload and heart failure.

A physician orders several drugs for a client with hemorrhagic stroke. Which drug order should the nurse question?

heparin sodium Explanation: Administering heparin, an anticoagulant, could increase the bleeding associated with hemorrhagic stroke. Therefore, the nurse should question this order to prevent additional hemorrhage in the brain. In a client with hemorrhagic stroke, the physician may use dexamethasone to decrease cerebral edema and pressure; methyldopa, to reduce blood pressure; and phenytoin, to prevent seizures.

A white male, age 43, with a tentative diagnosis of infective endocarditis is admitted to an acute care facility. His medical history reveals diabetes mellitus, hypertension, and pernicious anemia; he underwent an appendectomy 20 years earlier and an aortic valve replacement 2 years before this admission. Which history finding is a major risk factor for infective endocarditis?

history of aortic valve replacement Explanation: A heart valve prosthesis such as an aortic valve replacement is a major risk factor for infective endocarditis. Other risk factors include a history of heart disease (especially mitral valve prolapse), chronic debilitating disease, I.V. drug abuse, and immunosuppression. Although race, age, and a history of diabetes mellitus may predispose a person to cardiovascular disease, they aren't major risk factors for infective endocarditis.

Following a left anterior myocardial infarction, a client undergoes insertion of a pulmonary artery catheter. Which finding most strongly suggests left-sided heart failure?

increased pulmonary artery diastolic pressure Explanation: Increased pulmonary artery diastolic pressure suggests left-sided heart failure. Central venous pressure increases in heart failure rather than decreases. The cardiac index decreases in heart failure. The mean pulmonary artery pressure increases in heart failure.

The nurse is reviewing the Nurse's Notes and electrocardiogram (ECG) results of 0900.

loss of appetite and abdominal pain dizziness when standing tenting of skin when pinched Explanation: This older adult with atrial fibrillation, hypertension, and heart failure has multiple assessment findings that require follow-up. A loss of appetite and abdominal pain in a client with atrial fibrillation and heart failure, who is taking several medications, requires follow-up.The client has orthostatic hypotension, a drop in systolic blood pressure of 20 mm Hg or more or a drop in diastolic blood pressure of 10 mm Hg or more when changing from a lying to a standing position. This finding requires follow-up and can occur with dehydration. The tenting and slow return of the skin to normal when pinched is a sign of dehydration and requires follow-up. The client's heart rate is within normal range and does not require follow-up. The oxygen saturation level of 96% is within normal limits and does not require follow-up. The client's electrocardiogram (ECG) shows normal sinus rhythm and does not require follow-up.

Before surgery to repair an aortic aneurysm, the client's pulse pressure begins to widen, suggesting increased aortic valvular insufficiency. If the branches of the aortic arch are involved, the nurse should assess the client for:

loss of consciousness. Explanation: If the aortic arch is involved, there will be a decrease in the blood flow to the cerebrum. Therefore, loss of consciousness will be observed. A sudden loss of consciousness is a primary symptom of rupture and no blood flow to the brain.Anxiety is not a sign of aortic valvular insufficiency.The end result of decreased cerebral blood flow is loss of consciousness, not headache or disorientation.

A female client who is 32 years of age has been diagnosed with stage 1 hypertension. The client's height is 5 feet 5 inches (165 cm), and her weight is recorded as 125 pounds (56.6 kg); she reports that she frequently eats at "fast food" restaurants and enjoys a glass of wine to relax on weekends. In developing a teaching plan for this client, the nurse should address which topic?

low-sodium food choices Explanation: Lifestyle modification to lower blood pressure includes weight reduction in clients who are overweight, reducing the intake of dietary sodium, and an increase in physical activity. Client teaching involves instruction on low sodium diet and foods because of the propensity for high-sodium foods at fast food restaurants. The client is of a normal weight, and alcohol intake is in moderation. Nitroprusside is a treatment for hypertensive crisis.

A client has mitral stenosis and is a prospective valve recipient. The nurse is instructing the client about health maintenance prior to surgery. Inability to follow which of the following prescription would pose the greatest health hazard to this client at this time?

medication therapy Explanation: Preoperatively, anticoagulants may be prescribed for the client with advanced valvular heart disease to prevent emboli. Postoperatively, all clients with mechanical valves and some clients with bioprosthesis are maintained indefinitely on anticoagulant therapy. Adhering strictly to a dosage schedule and observing specific precautions are necessary to prevent hemorrhage or thromboembolism. Some clients are maintained on lifelong antibiotic prophylaxis to prevent recurrence of rheumatic fever. Episodic prophylaxis is required to prevent infective endocarditis after dental procedures or upper respiratory, gastrointestinal, or genitourinary tract surgery. Diet modification, activity restrictions, and dental care are important; however, they do not have as much significance postoperatively as medication therapy does.

The nurse is assessing the ankle-brachial index (ABI) for a client with peripheral vascular disease. The highest systolic pressure for each ankle is 80 mm Hg and the highest brachial pressure is 160 mm Hg. What does this client's ABI indicate?

mild to moderate insufficiency Explanation: ABI is calculated by dividing the highest systolic pressure for each ankle by the highest brachial pressure. For this client it would be 80/160 mm Hg = 0.50 ABI. This indicates that the client has mild to moderate insufficiency. Clients with ABI of about 1.0 have no arterial insufficiency; clients with ABI of less than 0.50 have ischemic rest pain; and clients with an ABI of 0.40 or less indicates severe ischemia or tissue loss.

A client has a coxsackie B (viral) or trypanosomal (parasite) infection. The nurse should further assess the client for which health problem?

myocarditis Explanation: Intracellular microorganisms, such as viruses and parasites, invade the myocardium to survive. These microorganisms damage the vital organelles and cause cell death in the myocardium. The myocardium becomes weak, leading to heart failure; then T lymphocytes invade the myocardium in response to the viral infection. The T lymphocytes respond to the viral infection by secreting cytokines to kill the virus, but they also kill the virus-infected myocardium. Myocardial infarction, renal failure, and liver failure are not direct consequences of a viral or parasitic infection.

A client is diagnosed with myocardial infarction. Which data collection findings indicate that the client has developed left-sided heart failure? Select all that apply.

orthopnea cough crackles Explanation: Left-sided heart failure produces primarily pulmonary signs and symptoms, such as orthopnea, cough, and crackles. Right-sided heart failure primarily produces systemic signs and symptoms, such as ascites, jugular vein distention, and hepatomegaly.

Which assessment finding(s) indicates an effective outcome? Select all that apply.

orthostatic blood pressure skin returns quickly when pinched client will use one pharmacy client verbalized the need to reduce potassium in diet serum digoxin level serum potassium level

A client has been diagnosed with peripheral arterial occlusive disease. In order to promote circulation to the extremities, the nurse should instruct the client to:

participate in a regular walking program. Explanation: Clients diagnosed with peripheral arterial occlusive disease should be encouraged to participate in a regular walking program to help develop collateral circulation. They should be advised to rest if pain develops and resume activity when pain subsides.With arterial disease, extremities should be kept in a dependent position to promote circulation; elevation of the extremities will decrease circulation.To avoid burns, heating pads should not be used by anyone with impaired circulation.Massaging the calf muscles will not decrease pain. Intermittent claudication subsides with rest.

A client with peripheral vascular disease has undergone a right femoral-popliteal bypass graft. The blood pressure has decreased from 124/80 mm Hg to 88/62 mm Hg. What should the nurse assess first?

pedal pulses Explanation: With each set of vital signs, the nurse should assess the dorsalis pedis and posterior tibial pulses. The nurse needs to ensure adequate perfusion to the lower extremity with the drop in blood pressure. IV fluids, nasal cannula setting, and capillary refill are important to assess; however, priority is to determine the cause of drop in blood pressure and that adequate perfusion through the new graft is maintained.

A client is recovering from an acute myocardial infarction (MI). During the first week of the client's recovery, the nurse should stay alert for which abnormal heart sound?

pericardial friction rub Explanation: A pericardial friction rub, which sounds like squeaky leather, may occur during the first week following an MI. Resulting from inflammation of the pericardial sac, this abnormal heart sound arises as the roughened parietal and visceral layers of the pericardium rub against each other. Certain stenosed valves may cause a brief, high-pitched opening snap heard early in diastole. Graham Steele murmur is a high-pitched, blowing murmur with a decrescendo pattern; heard during diastole, it indicates pulmonary insufficiency, such as from pulmonary hypertension or a congenital pulmonary valve defect. An ejection click, associated with mitral valve prolapse or a rigid, calcified aortic valve, causes a high-pitched sound during systole.

A nurse in the emergency department is caring for a client with acute heart failure. Which laboratory value is most important for the nurse to check before administering medications to treat heart failure?

potassium Explanation: Diuretics, such as furosemide, are commonly used to treat acute heart failure. Most diuretics increase the renal excretion of potassium. The nurse should check the client's potassium level before administering diuretics, and obtain an order to replace potassium if the level is low. Other medications commonly used to treat heart failure include angiotensin-converting enzyme inhibitors, digoxin, and beta-adrenergic blockers. Although checking the platelet count, calcium level, and WBC count are important, these values don't affect medication administration for acute heart failure.

A physician orders digoxin for a client with heart failure. During digoxin therapy, which laboratory value may predispose the client to digoxin toxicity?

potassium level of 3.1 mEq/L (3.1 mmol/L) Explanation: Conditions that may predispose a client to digoxin toxicity include hypokalemia (evidenced by a potassium level less than 3.5 mEq/L), hypomagnesemia (evidenced by a magnesium level less than 1.5 mEq/L), hypothyroidism, hypoxemia, advanced myocardial disease, active myocardial ischemia, and altered autonomic tone. Hypermagnesemia (evidenced by a magnesium level greater than 2.5 mEq/L), hypercalcemia (evidenced by an ionized calcium level greater than 5.3 mg/dl), and hypernatremia (evidenced by a sodium level greater than 145 mEq/L) aren't associated with a risk of digoxin toxicity.

A nurse is caring for a client who is on a continuous cardiac monitor. When evaluating the client's rhythm strip, the nurse notes that the QRS interval has increased from 0.08 second to 0.14 second. Based on this finding, the nurse should withhold continued administration of which drug?

procainamide Explanation: Procainamide may cause an increased QRS complexes and QT intervals. If the QRS duration increases by more than 50%, then the nurse should withhold the drug and notify the physician of the nurse's finding. Metoprolol may cause increased PR interval and bradycardia. Propafenone and verapamil may cause bradycardia and atrioventricular blocks.

A postoperative client is receiving heparin after developing thrombophlebitis. The nurse monitors the client carefully for bleeding and other adverse effects of heparin. If the client starts to exhibit signs of excessive bleeding, the nurse should expect to administer an antidote that is specific to heparin. Which agent fits this description?

protamine sulfate Explanation: Protamine sulfate is the antidote specific to heparin. Phytonadione (vitamin K) is the antidote specific to oral anticoagulants such as warfarin. (Heparin isn't given orally.) Thrombin is a hemostatic agent used to control local bleeding. Plasma protein fraction, a blood derivative, supplies colloids to the blood and expands plasma volume; it's used to treat clients who are in shock.

During handing-off communication, the nurse in the cardiac step-down unit learns that a client with a history of anterior wall myocardial infarction has had high normal pulmonary artery wedge pressures. For what should the nurse particularly assess in this client?

pulmonary crackles Explanation: High pulmonary artery wedge pressures are diagnostic for left-sided heart failure. With left-sided heart failure, pulmonary edema can develop and cause pulmonary crackles. In left-sided heart failure, hypotension may result. An elevated pulmonary artery wedge pressure will not affect respiratory rate. A productive cough (not a dry cough) may develop because of excess fluid in the pulmonary vasculature and tissues.

What is a priority nursing assessment in the first 24 hours after admission of the client with a thrombotic stroke?

pupil size and pupillary response Explanation: It is crucial to monitor the pupil size and pupillary response to indicate changes around the cranial nerves. The cholesterol level is not a priority assessment, although it may be an assessment to be addressed for long-term healthy lifestyle rehabilitation. Bowel sounds need to be assessed because an ileus or constipation can develop, but this is not a priority in the first 24 hours, when the primary concerns are cerebral hemorrhage and increased intracranial pressure. An echocardiogram is not needed for the client with a thrombotic stroke without heart problems.

The nurse on a medical unit is caring for a 61-year-old male client with multiple myeloma who presents with coughing, fever, and chills. The nurse is reviewing the Nurse's Notes and Laboratory Results of 0900, day 1, with the health care provider who has diagnosed the client with pneumonia. Complete the following sentence by choosing from the lists of options.

recurrent infections as evidenced by absolute lymphocyte count (ALC)

The nurse is assessing a client with superficial thrombophlebitis in the greater saphenous vein of the left leg. The client has "aching" in the leg. Which finding indicates the nurse should contact the health care provider (HCP) to request a prescription to improve the client's comfort?

red, warm, palpable linear cord along the vein that is painful on palpation Explanation: Superficial thrombophlebitis is associated with pain, warmth, and erythema. The nurse can request a prescription for warm packs to relieve the pain. Venous insufficiency causes edema and a brown discoloration of the lower leg. Varicose veins are dark, protruding veins, and symptoms of discomfort increase with standing. Pain on dorsiflexion of the foot indicates deep vein thrombosis; the client does not indicate having this pain.

A client is discharged to a heart rehabilitation program. What lifestyle changes would be appropriate for the nurse to review?

reducing cholesterol levels, increasing activity levels progressively, and coping strategies Explanation: Cardiac rehabilitation is designed to assist the client in regaining functioning gradually. It also includes heart-healthy information such as dietary changes, a progressive increase in activity, and effective coping strategies for stress reduction. The emphasis is on lifestyle changes and reducing the risk of recurrence. The information related to unsaturated fats and participation in burst training is inaccurate. There is no need to reduce calcium intake and sodium is not increased. Homocysteine levels should be decreased, not increased.

A client with a history of hypertension and peripheral vascular disease underwent an aortobifemoral bypass graft. Preoperative medications included pentoxifylline, metoprolol, and furosemide. On postoperative day 1, the 1200 vital signs are: temperature 98.9° F (37.2° C); heart rate 132 bpm; respiratory rate 20 breaths/min; blood pressure 126/78 mm hg. Urine output is 50 to 70 mL/h. The hemoglobin and the hematocrit are stable. The medications have not been prescribed for administration after surgery. Using the SBAR (Situation-Background-Assessment-Recommendation) technique for communication, the nurse contacts the health care provider (HCP) and recommends to:

restart the metoprolol. Explanation: The client is experiencing a rebound tachycardia from abrupt withdrawal of the beta blocker. The beta blocker should be restarted due to the tachycardia, history of hypertension, and the desire to reduce the risk of postoperative myocardial morbidity. The bypass surgery should correct the claudication and need for pentoxifylline. The furosemide and increase in fluids are not indicated since the client's urine output and blood pressure are satisfactory and there is no indication of bleeding. The nurse should also determine the potassium level before starting the furosemide.

The nurse is assessing a client who has a long history of uncontrolled hypertension. The nurse should assess the client for damage in which area of the eye?

retina Explanation: The retina is especially susceptible to damage in a client with chronic hypertension. The arterioles supplying the retina are damaged. Such damage can lead to vision loss. The iris, cornea, and sclera are not affected by hypertension.

A client is scheduled for an arteriogram. The nurse should explain to the client that the arteriogram will confirm the diagnosis of occlusive arterial disease by:

showing the location of the obstruction and the collateral circulation. Explanation: An arteriogram involves injecting a radiopaque contrast agent directly into the vascular system to visualize the vessels. It usually involves computed tomographic scanning.The velocity of the blood flow can be estimated by duplex ultrasound.The client's ankle-brachial index is determined, and then the client is requested to walk. The normal response is little or no drop in ankle systolic pressure after exercise.

Which position is best for a client with heart failure who has orthopnea?

sitting upright (high Fowler's position) with legs resting on the mattress Explanation: Sitting almost upright in bed with the feet and legs resting on the mattress decreases venous return to the heart, thus reducing myocardial workload. Also, the sitting position allows maximum space for lung expansion. Low Fowler's position would be used if the client could not tolerate high Fowler's position for some reason. Lying on the right side would not be a good position for the client in heart failure. The client in heart failure would not tolerate Trendelenburg position.

Which is the most appropriate diet for a client during the acute phase of myocardial infarction (MI)?

small, easily digested meals Explanation: Recommended dietary principles in the acute phase of MI include avoiding large meals because small, easily digested foods are better tolerated. Fluids are given according to the client's needs, and sodium restrictions may be prescribed, especially for clients with manifestations of heart failure. Cholesterol restrictions may be prescribed as well. Clients are not prescribed diets of liquids only or restricted to nothing by mouth unless their condition is very unstable.

Considering a client's atrial fibrillation, a nurse must administer digoxin with caution because it

stimulates the parasympathetic division of the autonomic nervous system, increasing vagal tone. Explanation: A nurse must administer digoxin with caution in a client with atrial fibrillation because digoxin stimulates the parasympathetic division of the autonomic nervous system, increasing vagal tone. The vagal effect slows the heart rate, increases the refractory period, and slows conduction through the atrioventricular node and junctional tissue, increasing the potential for new arrhythmias to develop. Digoxin doesn't constrict arteries. Although digoxin can trigger proarrhythmias, it does so by increasing vagal tone (not stroke volume).

Which action is most important for the nurse to perform post procedure in a client with impaired renal function who is scheduled for a multidetector-computed tomography (MDCT) to evaluate peripheral circulation?

strictly monitoring intake and output Explanation: After an MDCT procedure, clients with impaired renal function should be monitored closely for urine output of at least 0.5 mL/kg/hr because they are at risk for contrast-induced nephropathy. Before the procedure, there may be an indication for IV fluids and sodium bicarbonate to alkalinize urine and protect against free radical damage. Allergies should also be assessed prior to the procedure and treated with steroids and/or histamine blockers if necessary.

The nurse is caring for an older adult man who walks 2 miles every morning. The nurse notes that during his morning walk, he called his child and stated that he thought that he was having a heart attack. Which symptom, identified by the client, is the most common and consistent with that of a heart attack (myocardial infarction)?

substernal pain Explanation: The most common symptom in a male with a myocardial infarction is crushing substernal pain, resulting from deprivation of oxygen to the heart. Dyspnea is the second most common symptom, related to an increase in metabolic needs of the body. Edema is a later sign of heart failure, commonly seen later as the body tries to recover from the tissue damage. Palpitations may result from reduced cardiac output, producing arrhythmias.

A client with migraine headaches and a history of angina asks the nurse why the health care provider does not prescribe one of the newer medications for migraine, such as sumatriptan. The nurse responds that:

sumatriptan is contraindicated in clients with angina. Explanation: Sumatriptan is contraindicated in clients with ischemic heart disease, such as angina, myocardial infarction, or coronary artery disease, because it is a vasoconstrictor.The cost of the medication is not the concern at this time; the drugs are contraindicated because of the client's history of angina.Sumatriptan is used for the abortive treatment of migraines, not prophylactic treatment, and it is effective in treating acute migraines with or without aura.

Which set of postural vital signs in a client with hypertension should the nurse report to the health care provider (HCP)?

supine 120/70 mm Hg, 70 bpmsitting 102/64 mm Hg, 86 bpmstanding 100/60 mm Hg, 92 bpm Explanation: There was a significant change in both blood pressure and heart rate with position change, indicating inadequate blood volume to sustain normal values. The nurse should report this change to the HCP. Normal postural changes allow for an increase in heart rate of 5 to 20 bpm, a possible slight decrease of <5 mm Hg in the systolic blood pressure, and a possible slight increase of <5 mm Hg in the diastolic blood pressure.

What position should the nurse use for the client with venous insufficiency to enhance blood supply?

supine with lower extremities elevated Explanation: For clients with venous insufficiency, blood return to the heart needs to be enhanced; therefore, the nurse should position them in a position with lower extremities elevated. Prone with head turned to one side and Fowler with lower extremities in neutral position does not elevate the extremities and, therefore, does not increase blood supply. Dorsal recumbent with legs separated is used for special situations, not to enhance blood supply in venous insufficiency.

A client who had an exploratory laparotomy 3 days ago now has a white blood cell (WBC) count of 15,000 µL (15x109/L). For which clinical findings of this laboratory report should the nurse assess the client? Select all that apply.

swelling around the incision redness around the incision elevated temperature The client has an elevated white count. Normal white count is 4,300 to 10,800 µL (4.3 to 10.8 x 109/L). The client is at risk for infection, and the nurse should assess the client for inflammation around the incision site, redness at the incision site, and elevated temperature. The client should be encouraged to cough and deep breathe, and it is unlikely that a cough is related to an incisional infection. Weak pedal pulses are not indications of an infection, but the nurse should report this finding if it persists.

On a routine visit to the health care provider, a client with chronic arterial occlusive disease reports quitting smoking after 34 years. To relieve symptoms of intermittent claudication, a condition associated with chronic arterial occlusive disease, which additional measure should the nurse recommend?

taking daily walks Explanation: Taking daily walks relieves symptoms of intermittent claudication, although the exact mechanism is unclear. Anaerobic exercise may make these symptoms worse. Clients with chronic arterial occlusive disease must reduce daily fat intake to 30% or less of total calories. The client should limit dietary cholesterol because hyperlipidemia is associated with atherosclerosis, a known cause of arterial occlusive disease. However, HDLs have the lowest cholesterol concentration, so this client should eat, not abstain from, foods that raise HDL levels.

Which client has a need for prophylactic antibiotic therapy prior to dental manipulations?

the client who had an aortic valve replacement 5 years ago Explanation: A heart valve prosthesis such as an aortic valve replacement is a major risk factor for the development of infective endocarditis. Preventative measures include antibiotic prophylaxis prior to dental work. Other implanted devices (hip, knee, ICD) can increase the risk of infection, but the client with the greatest risk is the one with the valve replacement.

The nurse is preparing the client with a cerebrovascular accident for discharge to home. Which will influence the client's continuing progress in rehabilitation at home?

the family's ability to provide support to the client Explanation: The strong support of family members is frequently identified as an important factor that influences a cerebrovascular accident client's continuing progress in rehabilitation after discharge. Discharge planning should prepare the client and family for the many changes necessary when the client returns home.Continuing progress in rehabilitation is not dependent upon the client's ability to ambulate.A client's continuing progress in rehabilitation is more dependent on the client and family support than the home health aide.Follow-up visits with the health care provider are important but are not the greatest influence on the client's progress with rehabilitation.

The nurse connects a client to the electrocardiogram (EKG) monitor. The nurse would plan the need for transcutaneous pacing with observation of which heart rhythm?

third-degree atrioventricular block Explanation: The nurse would retrieve the defibrillator, which has the capability of transcutaneous pacing, when observing a third-degree atrioventricular block. There is no communication between the atria and ventricles with this block, therefore the client would be bradycardiac with a reduced cardiac output. Sinus bradycardia may be a normal finding based on the client and would not be automatically treated with transcutaneous pacing. Having premature junctional contractions with a normal sinus rhythm will continue to produce a normal cardiac output due to the rate of 60-100 beats/minute. With ventricular asystole, cardiopulmonary resuscitation is needed instead.

The nurse is developing a discharge teaching plan for a client who had a graft insertion for an abdominal aortic aneurysm 4 days ago. The nurse reviews the client's chart for information about the client's history. See the chart. Based on the data and expected outcomes, which area should the nurse emphasize in the teaching plan?

tissue perfusion Explanation: The underlying pathophysiology in this client is atherosclerosis. The findings from the assessment indicate the risk factors of smoking and high blood pressure. Therefore, tissue perfusion is a priority for health promoting education. The data do not support education that focuses on food or fluid intake. Although edema is a potential problem and could contribute to poor skin integrity, the edema will likely be resolved by the aneurysm repair.

The nurse is caring for a client post myocardial infarction (MI). Orders include strict bed rest and a clear, liquid diet. What is the nurse's best response to the client who is inquiring about the purpose of the new diet?

to reduce the metabolic workload of digestion Explanation: Acute care of the client with an MI is aimed at reducing the cardiac workload. Clear liquids are easily digested to help reduce this workload. Sympathetic nervous system involvement causes decreased peristalsis and gastric secretion, so limiting food intake helps prevent gastric distension and cardiac workload. A clear diet will not reduce gastric acidity or blood glucose, and fecal elimination will still occur, so these are incorrect choices.

A client is discharged after an aortic aneurysm repair with a synthetic graft to replace part of the aorta. The nurse should instruct the client to notify the health care provider (HCP) before having which procedure?

tooth extraction Explanation: The client with a synthetic graft may need to be treated with prophylactic antibiotics before undergoing major dental work and should notify the HCP before any such procedure. Prophylactic antibiotic treatment reduces the danger of systemic infection caused by bacteria from the oral cavity. Venous access for drawing blood, IV line insertion, and X-rays do not contribute to the risk of infection.

While transferring a load of firewood from the front driveway to the backyard woodpile at 11 a.m., the client experienced a heaviness in the chest and dyspnea. The client stopped working and rested, and the pain subsided. At noon, the pain returned. At 12:30 p.m., the client's spouse took the client to the emergency department. Around 1:30 p.m., the health care provider diagnosed an anterior myocardial infarction (MI). The nurse should anticipate which orders by the health care provider?

ublingual nitroglycerin, tissue plasminogen activator (tPA), and telemetry Explanation: The nurse should anticipate an order for sublingual nitroglycerin, tPA, and telemetry. The tPA is appropriate because the client's chest pain began less than 3 hours before diagnosis. The tPA is more specific for cardiac tissue than streptokinase. Stress testing should not be performed during an MI. The client does not exhibit symptoms that indicate the use of lidocaine. Lidocaine is usually used as an anesthetic and is also used for acute treatment of ventricular arrhythmia, but anterior myocardial infarction results from reduction in blood supply to the anterior wall of the heart due to coronary artery occlusion.

The nurse is monitoring a client admitted with a myocardial infarction (MI) who is at risk for cardiogenic shock. The nurse should report which change on the client's chart to the health care provider?

urine output Explanation: Oliguria occurs during cardiogenic shock because there is reduced blood flow to the kidneys. Typical signs of cardiogenic shock include low blood pressure, rapid and weak pulse, decreased urine output, and signs of diminished blood flow to the brain, such as confusion and restlessness. Cardiogenic shock is a serious complication of MI, with a mortality rate approaching 90%. Fever is not a typical sign of cardiogenic shock. The other changes in vital signs on the client's chart are not as significant as the decreased urinary output.

The client returns to the hospital 3 days after diagnosis of deep vein thrombosis, with reports of cough, hemoptysis, shortness of breath, and sharp pain under the right scapula. The client is subsequently is diagnosed with a pulmonary embolus (PE). The client asks the nurse, "How did I even get a pulmonary embolus?" What is the best response by the nurse? Select all that apply.

venous endothelial changes having any condition that produces venous stasis increased blood coagulability Explanation:The nurse should relay to the client that a pulmonary embolism is caused by blood clots that travel to the lungs from the legs or, rarely, other parts of the body. Because the clots block blood flow to the lungs, a pulmonary embolism can be life-threatening. Major risk factors for the development of PE include any condition that produces venous stasis, increased blood coagulability, or venous endothelial changes. Major risk factors do not include frequent falls or taking medications such as warfarin sodium and should not be communicated to the client.


Related study sets

PHS semester examTyson has decided he wants to be trained and certified in cosmetology. Which type of institution of higher learning could provide him with this?

View Set

Chapter 16: Documenting, Reporting, Conferring, and Using Informatics

View Set

1-36 Elements + Sr, Ba, Sn, Pb, I, Xe, Rn

View Set

Google Cloud Certified Professional Cloud Architect - quiz#1

View Set

RETIREMENT PLANS: VARIABLE ANNUITIES (UITS)

View Set